Women's Health Final Exam Tuesday 6/6/2017 [60 qs]

¡Supera tus tareas y exámenes ahora con Quizwiz!

*preterm labor (PTL)*

*20-37 weeks* the most serious complication of pregnancy 2nd only to congenital anomalies as leading causes of infant mortality in the US it is the result of many things not just 1 factor

*mgmt of hyperemesis gravidarum*

*Priority Interventions: it's is nausea and vomiting in early pregnancy that prevents the woman from ingesting adequate nutrition. IV fluids may be required for rehydration, but the priority is to stop all intake of food and fluid for a period of time until vomiting has stopped.* - Hyperemesis gravidarum is a diagnosis of exclusion. Careful consideration of other conditions must be assessed when a client experiences nausea and vomiting for the first time after 9 weeks' gestation. Conservative management in the home is the first line of treatment for the woman with hyperemesis gravidarum. This usually focuses on dietary and lifestyle changes. If conservative management fails to alleviate the client's symptoms and nausea and vomiting continue, hospitalization is necessary to reverse the effects of severe nausea and vomiting. On admission to the hospital, blood tests are ordered to assess the severity of the client's dehydration, electrolyte imbalance, ketosis, and malnutrition. Parenteral fluids and drugs are ordered to rehydrate the woman and reduce the symptoms. The first choice for fluid replacement is generally normal saline, which aids in preventing hyponatremia, with vitamins (pyridoxine [B6]) and electrolytes added. Oral food and fluids are withheld for the first 24 to 36 hours to allow the gastrointestinal tract to rest. Antiemetics may be administered rectally or intravenously to control the nausea and vomiting initially because the woman is considered NPO (not able to ingest anything by mouth). Once her condition stabilizes and she is allowed oral intake, medications may be administered orally. If the client does not improve after several days of bed rest, "gut rest," IV fluids, and antiemetics, total parenteral nutrition or feeding through a percutaneous endoscopic gastrostomy tube is instituted to prevent malnutrition. Administering antiemetics intravenously or intramuscularly is typically the second pillar of treatment for hyperemesis gravidarum. Finding a drug that works for any given client is largely a matter of trial and error. If one drug is ineffective, another class of drugs with a different mechanism of action may help. Promethazine (Phenergan) and prochlorperazine (Compazine) are among the older preparations usually tried first. If they fail to relieve symptoms, newer drugs such as ondansetron (Zofran) may be tried. Most drugs are given intravenously or intramuscularly. There is no evidence that any antiemetic class is superior to another with respect to effectiveness - Few women receive complete relief of symptoms from any one therapy. Complementary and alternative medicine therapies appeal to many women as supplements to traditional ones. Some popular therapies include acupressure, massage, therapeutic touch, ginger, and the wearing of Sea-Bands to prevent nausea and vomiting. Recent research has reported a positive effect of using acupressure (provided by Sea-Bands) over the nei guan acupoint on the wrist to control nausea and vomiting associated with pregnancy

*clinical manifestations of hyperemesis gravidarum*

- Begin the history by asking the client about the onset, duration, and course of her nausea and vomiting. Ask her about any medications or treatments she used and how effective they were in relieving her nausea and vomiting. Obtain a diet history from the client, including a dietary recall for the past week. Note the client's knowledge of nutrition and need for appropriate nutritional intake. Be alert for patterns that may contribute to or trigger her distress. Also ask about any complaints of ptyalism (excessive salivation), anorexia, indigestion, and abdominal pain or distention. Ask if she has noticed any blood or mucus in her stool. Review the client's history for possible risk factors, such as young age, nausea and vomiting with previous pregnancy, history of intolerance of oral contraceptives, nulliparity, trophoblastic disease, multiple gestation, emotional or psychological stress, gastroesophageal reflux disease, primigravida status, obesity, hyperthyroidism, and Helicobacter pylori seropositivity. Weigh the client and compare this weight with her weight before she began experiencing symptoms and to her prepregnancy weight to estimate the degree of loss. With hyperemesis, weight loss usually exceeds 5% of body mass. Inspect the mucous membranes for dryness and check skin turgor for evidence of fluid loss and dehydration. Assess blood pressure for changes, such as hypotension, that may suggest a fluid volume deficit. Also note any complaints of weakness, fatigue, activity intolerance, dizziness, or sleep disturbances. Assess the client's perception of the situation. Note any evidence of depression, anxiety, irritability, mood changes, and decreased ability to concentrate, which can add to her emotional distress. Much of the psychological distress is self-limiting in this condition and probably in the causal pathway. Determine the woman's support systems that are available for help. - Liver enzymes—to rule out hepatitis, pancreatitis, and cholestasis; elevations of aspartate aminotransferase (AST) and alanine aminotransferase (ALT) are usually present CBC—elevated levels of red blood cells and hematocrit, indicating dehydration Urine ketones—positive when the body breaks down fat to provide energy in the absence of adequate intake TSH and T4 to rule out thyroid disease Blood urea nitrogen—increased in the presence of salt and water depletion Urine specific gravity—greater than 1.025, possibly indicating concentrated urine linked to inadequate fluid intake or excessive fluid loss; ketonuria Serum electrolytes—decreased levels of potassium, sodium, and chloride resulting from excessive vomiting and loss of hydrochloric acid in stomach Ultrasound—evaluation for molar pregnancy or multiple gestation

*cardio changes during pregnancy*

- increase CO & HR - increase BV - increase blood clotting factors - Supine hypotension - Cardiac hypertrophy

*mgmt of placenta previa*

-*Take Note! Avoid doing vaginal examinations in the woman with placenta previa because they may disrupt the placenta and cause hemorrhage.* - Therapeutic management depends on the extent of bleeding, the amount of placenta over the cervical os, whether the fetus is developed enough to survive outside the uterus, the position of the fetus, the mother's parity, and the presence or absence of labor. With the increase in the rate of previous cesarean sections, the frequency of placenta previa has increased. Most women continue to present in emergency departments, therefore the associated morbidity due to hemorrhage remains high. Efforts should be made to avoid primary cesarean section where possible. In addition, prenatal care and timely diagnosis of placenta previa on ultrasound can decrease the associated morbidity. If the mother and fetus are both stable, therapeutic management may involve expectant ("wait-and-see" or watchful waiting) care. This care can be carried out at home or on an antepartal unit in the health care facility. If there is no active bleeding and the client has ready access to reliable transportation, can maintain bed rest at home, and can comprehend instructions, expectant care at home is appropriate. However, if the client requires continuous care and monitoring and cannot meet the home care requirements, the antepartal unit is the best environment. - Whether the care setting is in the client's home or in the health care facility, the nurse focuses on monitoring the maternal-fetal status, including assessing for signs and symptoms of vaginal bleeding and fetal distress and providing support and education to the client and her family, including providing information about the diagnostic studies and procedures that are performed. For the majority of women, a cesarean birth will be planned. - Assess the degree of vaginal bleeding; inspect the perineal area for blood that may be pooled underneath the woman. Estimate and document the amount of bleeding. Perform a peripad count on an ongoing basis, making sure to report any changes in amount or frequency to the health care provider. If the woman is experiencing active bleeding, prepare for blood typing and cross-matching in the event a blood transfusion is needed. - Monitor maternal vital signs and uterine contractility frequently for changes. Have the client rate her level of pain using an appropriate pain rating scale. Assess fetal heart rates via Doppler or electronic monitoring to detect fetal distress. Monitor the woman's cardiopulmonary status, reporting any difficulties in respirations, changes in skin color, or complaints of difficulty breathing. Have oxygen equipment readily available should fetal or maternal distress develop. Encourage the client to lie on her side to enhance placental perfusion. If the woman has an intravenous (IV) line inserted, inspect the IV site frequently. Alternately, anticipate the insertion of an intermittent IV access device such as a saline lock, which can be used if quick access is needed for fluid restoration and infusion of blood products. Obtain laboratory tests as ordered, including complete blood count (CBC), coagulation studies, and Rh status if appropriate. Administer pharmacologic agents as necessary. Give Rh immunoglobulin if the client is Rh negative at 28 weeks' gestation. Monitor tocolytic (anticontraction) medication if prevention of preterm labor is needed.

*Nursing Management R/T NAS*

1. *PROMOTING COMFORT*: Supportive interventions to promote comfort include swaddling, low lighting, gentle handling, quiet environment with minimal stimulation, use of soft voices, pacifiers to promote "self-soothing," frequent small feedings, vertical rocking during infant disorganization periods, and rooming-in and positioning. Keep environmental stimuli to a minimum. For example, decrease stimuli by dimming the lights in the nursery, and swaddle the newborn tightly to decrease irritability behaviors. Other techniques such as gentle rocking, using a flexed position, and offering a pacifier can help manage CNS irritability. A pacifier also helps satisfy the newborn's need for nonnutritive sucking. Swaddling, pacifiers, low lighting, oscillating cribs, and avoidance of abrupt changes in the infant's environment can be helpful. Use a calm, gentle approach when handling the newborn and plan activities to avoid overstimulating the newborn, allowing time for rest periods. 2. *MEETING NUTRITIONAL NEEDS*: Newborns suffering from NAS have impaired feeding behaviors, such as excessive sucking, poor feeding, regurgitation, and diarrhea, which may cause weight loss. To improve weight gain, they are supplemented with high-calorie formula. When feeding the newborn, use small amounts and position the newborn upright to prevent aspiration and to facilitate rhythmic sucking and swallowing. Frequent small feedings are preferable and should provide 150 to 250 kcal/kg per 24 hours for proper growth of the infant undergoing significant withdrawal. Breast-feeding is encouraged unless the mother is still using drugs. Monitor the newborn's weight daily to evaluate the success of food intake. Assess hydration; check skin turgor and fontanels. Assess the frequency and characteristics of bowel movements and monitor the newborn's fluid and electrolyte and acid-base status. 3. *PREVENTING COMPLICATIONS*: Pharmacologic treatment is warranted if conservative measures, such as swaddling and decreased environmental stimulation, are not adequate. The AAP recommends that for newborns with confirmed drug exposure, drug therapy is indicated if the newborn has seizures, diarrhea, and vomiting resulting in excessive weight loss and dehydration, poor feeding, inability to sleep, and fever unrelated to infection. Common medications used in the management of newborn withdrawal include an opioid (morphine or methadone) and phenobarbital as a second drug if the opiate does not adequately control symptoms. Administer the prescribed medications and document the newborn's behavioral responses. The newborn is at risk for skin breakdown. Weight loss, diarrhea, dehydration, and irritability can contribute to this risk. Provide meticulous skin care and protect the newborn's elbows and knees against friction and abrasions. 4. *PROMOTING PARENT-NEWBORN INTERACTION*: For a mother who abuses substances, the birth of a drug-exposed newborn is both a crisis and an opportunity. The mother may feel guilty about the newborn's condition. Many of these newborns are unresponsive and have disorganized sleeping and feeding patterns. When awake, they can be easily overstimulated and irritated. Such characteristics make parent-newborn interactions difficult and frustrating, leading to possible detachment and avoidance. Nursing support, which includes a description of symptoms and their management, is vital if maternal-infant attachment is to occur and potential neglect or abuse is avoided. Instruct the mother or caretaker how to care for the newborn, including what to do after the newborn goes home. On the other hand, the newborn may be a powerful motivator for the mother to undergo treatment and seek recovery. Refer the mother to community agencies to address addiction and the infant's developmental needs. The nurse can play a pivotal role in assisting her to abstain from drug use and to promote effective parenting skills.

*current interventions to treat PTL!*

1. education - bed rest 2. medications - tocolytics (Magnesium Sulfate, Indomethacin, Nifedipine (Procardia), *terbutaline (Brethine)*) & glucocorticoids (betamethasone (Celestone)) 3. dietary needs

*Patient Teaching: Mastectomy* [10]

1. Elevate arm above the level of the heart 2. *Avoid BP, IV & blood draws!* 3. Avoid lifting >10lb 4. Avoid wearing tight elastic, clothing, or jewelry 5. Perform arm exercises 6. Wear sunscreen SPF ≥ 15 7. *Wear insect repellent to prevent bites* 8. Wear gloves for gardening 9. Use oven mitt to remove objects from oven 10. *Use electric razor for shaving, avoid cutting cuticles*

*Review the risks factors for male infertility.* *Risk factors for infertility in men include:* [12]

1. Exposure to toxic substances (lead, mercury, x-rays, chemotherapy) 2. *Cigarette or marijuana smoke* (may affect and or reduce the quality of sperm counts) 3. Heavy alcohol consumption 4. Use of prescription drugs for ulcers or psoriasis 5. Exposure of the genitals to high temperatures (hot tubs or saunas) 6. Hernia repair 7. Obesity is associated with decreased sperm quality 8. Cushing syndrome 9. Frequent long-distance cycling or running 10. STIs 11. Undescended testicles (cryptorchidism)- Hypospadias, Varicocele (varicose vein of the scrotum), Low testosterone levels 12. Mumps after puberty

PTL S/S

Cramping/Contractions Suprapubic pain or pressure Urinary frequency Vaginal discharge Rupture of membranes

*Know what alpha fetoprotein is and why it is tested for in pregnancy.* blood test Made in the liver of an unborn fetus Looking for: *spina bifida, encephalopathy, chromosome disorders/abnormalities* *Maternal serum alpha-fetoprotein screening (MSAFP)* (6) (2)

A sample of the woman's blood is drawn to evaluate *plasma protein* that is produced by the fetal liver, yolk sac, and GI tract, and crosses from the amniotic fluid into the maternal blood • typically performed b/w *15-18 weeks gestation* INCREASED levels might indicate a: 1. *neural tube defect* 2. *Turner syndrome* 3. *tetralogy of Fallot* 4. *multiple gestation* 5. *omphalocele gastroschisis* 6. *hydrocephaly* DECREASED levels might indicate 1. *Down syndrome* 2. *trisomy 18*

The nurse is measuring a contraction from the beginning of the increment to the end of the decrement for the same contraction. The nurse would document this as which finding? a. Duration b. Intensity c. Frequency d. peak

Duration

Hypoglycemia

Glucose is the major source of energy for organ function Typical characteristics: • Poor feedings • Jitteriness • Lethargy • High-pitched or weak cry • Apnea • Cyanosis and seizures Some newborns are asymptomatic

*mgmt of eclampsia*

Severe preeclampsia may develop suddenly and bring with it high blood pressure of more than 160/110 mm Hg, proteinuria of more than 5 g in 24 hours, oliguria of less than 400 mL in 24 hours, cerebral and visual symptoms, and rapid weight gain. This clinical picture signals severe preeclampsia, and immediate hospitalization is needed. Treatment is highly individualized and based on disease severity and fetal age. Birth of the infant is the only cure, because preeclampsia depends on the presence of trophoblastic tissue. Therefore, the exact age of the fetus is assessed to determine viability. Severe preeclampsia is treated aggressively because hypertension poses a serious threat to mother and fetus. The goal of care is to stabilize the mother-fetus dyad and prepare for birth. Therapy focuses on controlling hypertension, preventing seizures, preventing long-term morbidity, and preventing maternal, fetal, or newborn death. Intense maternal and fetal surveillance starts when the mother enters the hospital and continues throughout her stay. The woman in labor with severe preeclampsia typically receives oxytocin to stimulate uterine contractions and magnesium sulfate to prevent seizure activity. Oxytocin and magnesium sulfate can be given simultaneously via infusion pumps to ensure both are administered at the prescribed rate. Magnesium sulfate is given intravenously via an infusion pump. A loading dose of 4 to 6 g is given over 5 minutes. Then, a maintenance dose of 2 g/hr is given. The client is evaluated closely for magnesium toxicity. If at all possible, a vaginal delivery is preferable to a cesarean birth for better maternal outcomes and less risk associated with a surgical birth. PGE2 gel may be used to ripen the cervix. A cesarean birth may be performed if the client is seriously ill. A pediatrician/neonatologist or neonatal nurse practitioner should be available in the birthing room to care for the newborn. A newborn whose mother received high doses of magnesium sulfate needs to be monitored for respiratory depression, hypocalcemia, and hypotonia. Decreased fetal heart rate variability may occur but, in general, magnesium sulfate does not pose a risk to the fetus. The newborn may exhibit respiratory depression, loss of reflexes, muscle weakness, and neurologic depression - As with any seizure, the initial management is to clear the airway and administer adequate oxygen. Positioning the woman on her left side and protecting her from injury during the seizure are key. Suction equipment must be readily available to remove secretions from her mouth after the seizure is over. IV fluids are administered after the seizure at a rate to replace urine output and additional insensible losses. Fetal heart rate is monitored closely. Magnesium sulfate is administered intravenously to prevent further seizures. Serum magnesium levels, respiratory rate, reflexes, and urine output in women receiving magnesium sulfate are closely monitored to avoid magnesium toxicity and prevent cardiac arrest. Calcium gluconate (1 g intravenously) is typically ordered to counteract magnesium toxicity. Hypertension is controlled with antihypertensive medications. After the woman's seizures are controlled, her stability is assessed. If she is found stable, birth via induction or cesarean birth is performed. If the woman's condition remains stable, she will be transferred to the postpartum unit for care. If she becomes unstable after giving birth, she may be transferred to the critical care unit for closer observation.

*health hx and physical exam for preeclampsia*

Take a thorough history during the first antepartal visit to identify whether the woman is at risk for preeclampsia. Risk factors include: Primigravida status Chromosomal abnormalities Structural congenital anomalies Multiple gestation History of preeclampsia in a previous pregnancy Excessive placental tissue, as is seen in women with GTD Chronic stress Use of ovulation drugs Family history of preeclampsia (mother or sister) Lower socioeconomic status History of diabetes, hypertension, or renal disease Poor nutrition Lower socioeconomic status African-American ethnicity Age extremes (younger than 20 or older than 35) Obesity - In addition, complete a nutritional assessment that includes the woman's usual intake of protein, calcium, daily calories, and fluids. Women at risk for preeclampsia require more frequent prenatal visits throughout their pregnancy, and they require teaching about problems so that they can report them promptly. Blood pressure must be measured carefully and consistently. Obtain all measurements with the woman in the same position (blood pressure is highest in the sitting position and lowest in the side-lying position) and by using the same technique (automated vs. manual). This standardization in position and technique will yield the most accurate readings. Obtain the client's weight (noting gain since last visit), and assess for amount and location of edema. Asking questions such as "Do your rings still fit on your fingers?" or "Is your face puffy when you get up in the morning?" will help to determine whether fluid retention is present or if the woman's status has changed since her last visit. - *Take Note! Although edema is not a cardinal sign of preeclampsia, weight should be monitored frequently to identify sudden gains in a short time span. Current research relies less on the classic triad of symptoms (hypertension, proteinuria, and edema or weight gain) and more on decreased organ perfusion, endothelial dysfunction (capillary leaking and proteinuria), and elevated blood pressure as key indicators* - If edema is present, assess the distribution, degree, and pitting. Document your findings and identify whether the edema is dependent or pitting. Dependent edema is present on the lower half of the body if the client is ambulatory, where hydrostatic pressure is greatest. It is usually observed in the feet and ankles or in the sacral area if the client is on bed rest. Pitting edema is edema that leaves a small depression or pit after finger pressure is applied to a swollen area. Record the depth of pitting demonstrated when pressure is applied. Although subjective, the following is used to record relative degrees: 1+ pitting edema = 2-mm depression into skin; disappears rapidly 2+ pitting edema = 4-mm skin depression; disappears in 10 to 15 seconds 3+ pitting edema = 6-mm depression into skin; lasts more than 1 minute 4+ pitting edema = 8-mm depression into skin; lasts 2 to 3 minutes At every antepartal visit, assess the fetal heart rate with a Doppler device. Also check a clean-catch urine specimen for protein using a dipstick. - *Take Note! The absolute blood pressure (value that validates elevation) of 140/90 mm Hg should be obtained on two occasions 4 to 6 hours apart to be diagnostic of preeclampsia. Proteinuria is defined as 300 mg or more of urinary protein per 24 hours or more than 1+ protein by chemical reagent strip or dipstick of at least two random urine samples collected at least 4 to 6 hours apart with no evidence of urinary tract infection*

VEAL CHOP

Variable → Cord compression Early → Head compression Accelerations → Okay Late → Placenta

*current interventions for preterm birth!*

bedrest cessation of all but most basic needs specimens collected by the woman at home every 2 weeks for ~10 weeks

*clinical manifestations of HELLP*

similar to severe preeclampsia - Be alert for complaints of: nausea (with or without vomiting) (50%) malaise (90%) epigastric or RUQ pain (65%) demonstrable edema - diagnosis is made based on laboratory test results, including: Low hematocrit that is not explained by any blood loss Elevated LDH (liver impairment) Elevated AST (liver impairment) Elevated ALT (liver impairment) Elevated BUN Elevated bilirubin level Elevated uric acid and creatinine levels (renal involvement) Low platelet count (less than 100,000 cells/mm3)

*Review prenatal teaching with a patient with systemic lupus erythematosus (SLE).* Tidwell, Antepartum Complications

• The nurse should discuss with the woman the importance of having good control over her SLE condition throughout the pregnancy. • Discussions should focus on the effects of SLE during the pregnancy and possible risk for exacerbations. • Emphasize the importance of frequent prenatal visits to detect early preeclampsia, preterm labor, or infections. Instruction should cover the implications and potential side effects of all drug therapies prescribed. • Teach energy conservation techniques to prevent fatigue, signs and symptoms to report (extreme fatigue, edema, confusion, abdominal pain, weight loss, leg pain, anorexia), and the need for frequent and close monitoring for fetal well-being. • After childbirth, a discussion on birth control and the effects of the various methods on the disease is essential. • Referral to self-help groups and local and national SLE organizations is important for further education of the woman and her family. • SLE can greatly complicate a pregnancy if close supervision is not maintained. • The keys to a successful outcome for the mother and her infant include an accurate assessment of the disease and of the various systems involved, and vigilance during the pregnancy for disease progression, effects on the fetus, and development of complications. • Nursing care should be directed at early detection of problematic s/s, education of the mother and family, careful evaluation of the fetal status, and providing support to assist the mother in strengthening her coping strategies.

*Explain milk let-down reflex and what hormone is responsible.* Oxytocin: milk let down or ejection reflex Prolactin: milk production

• Typically, during the first 2 days after birth, the breasts are soft and nontender. • The woman also may report a tingling sensation in both breasts, which is the "let-down reflex" that occurs immediately before or during breast-feeding. • After this time, breast changes depend on whether the mother is breast-feeding or taking measures to prevent lactation. • During the postpartum period, oxytocin stimulates the uterus to contract during the breast-feeding session and for as long as 20 minutes after each feeding. • Oxytocin also acts on the breast by eliciting the milk let-down reflex during breast-feeding. • Prolactin is also associated with the breast-feeding process by stimulating milk production. • In women who breast-feed, prolactin levels remain elevated into the sixth week after birth • The levels of prolactin fluctuate in proportion to nipple stimulation. • Prolactin levels decrease in nonlactating women, reaching prepregnant levels by the third postpartum week. • High levels of prolactin have been found to delay ovulation by inhibiting ovarian response to follicle-stimulating hormone

*Describe the signs and symptoms of a prenatal patients diagnosed with Class I- IV cardiac diseases.* Tidwell, Antepartum Complications A woman's ability to function during the pregnancy is often more important than the actual diagnosis of the cardiac condition. The following is a functional classification system developed by the Criteria Committee of the New York Heart Association (1994) based on past and present disability and physical signs. It provides a simple way of classifying the extent of heart failure by placing people in 1 of 4 categories based on how much they are limited during physical activity, normal breathing, and varying degrees of SOB and/or chest pain:

*Class I*: asymptomatic with no limitation of physical activity; no objective evidence of cardiac disease. Ordinary physical activity does NOT cause undue fatigue, palpitation, dyspnea, or chest pain. *Class II*: symptomatic (dyspnea, chest pain) with increased activity resulting in slight limitation of physical activity. They are comfortable at rest. Ordinary physical activity results in fatigue, palpitation, dyspnea, or anginal pain. Minimal CVD present. *Class III*: symptomatic (fatigue, palpitations) with normal activity resulting in marked limitation of physical activity. They are comfortable at rest. Less than ordinary activity causes fatigue, palpitation, dyspnea, or anginal pain. Moderately severe CVD present. *Class IV*: symptomatic at rest or with any physical activity resulting in inability to carry on any physical activity without discomfort. Symptoms of heart failure or the anginal syndrome may be present even at rest. If any physical activity is undertaken, discomfort is increased. Severe CVD present.

*serum alpha-fetoprotein (AFP)* aka "Quad or Triple Screen" maternal assay

*Screening tool for neural tube defects, spina bifida, & chromosomal abnormalities* • Performed at 16-18wks gestation • Elevated AFP is associated with neural tube defects • Lower AFP is associated with down syndrome or other autosomal trisomies • Optional

*What is the role of pulmonary surfactant?* look at notes to see if there is more stuff

*a surface tension-reducing lipoprotein found in the newborn's lungs that prevents alveolar collapse at the end of expiration and loss of lung volume* • It lines the alveoli to enhance aeration of gas-free lungs, thus reducing surface tension and lowering the pressure required to open the alveoli. • Normal lung function depends on surfactant, which permits a decrease in surface tension at end expiration (to prevent atelectasis) and an increase in surface tension during lung expansion (to facilitate elastic recoil on inspiration). • It provides the lung stability needed for gas exchange. • The newborn's first breath, in conjunction with surfactant, overcomes the surface forces to permit aeration of the lungs. • The chest wall of the newborn is floppy because of the high cartilage content and poorly developed musculature. • Thus, accessory muscles to help in breathing are ineffective. possible complication of the neonate: *Respiratory distress syndrome* resulting from poor surfactant production secondary to hyperinsulinemia inhibiting the production of phospholipids, which make up surfactant - Fetus will have nasal flaring, grunting, just a lot of trouble breathing after birth breathing

*nursing assessments of postpartum psychosis* At the severe end of the continuum of postpartum emotional disorders is postpartum psychosis, which occurs in 1 in 1,000 live births

*an emergency psychiatric condition, can result in a significant increased risk for suicide and infanticide* • S/s: *mood lability, delusional beliefs, hallucinations, and disorganized thinking*, can be frightening for the women who are affected and for their families • generally surfaces w/i 3 months of giving birth and is manifested by sleep disturbances, fatigue, depression, and hypomania. • The mother will be tearful, confused, and preoccupied with feelings of guilt and worthlessness. • Early s/s resemble those of depression, but they may escalate to delirium, hallucinations, extreme disorganization of thought, anger toward herself and her infant, bizarre behavior, delusions, disorientation, depersonalization, delirium-like appearance, manifestations of mania, and *thoughts of hurting herself and the infant.* • mother frequently loses touch with reality and experiences a severe regressive breakdown, associated with a high risk of *suicide or infanticide* • *should NOT be left alone with their infants* • most women are *hospitalized for up to several months* • Psychotropic drugs are almost always part of treatment, along with individual psychotherapy and support group therapy. • Anti-psychotics and lithium are drugs of choice if not breastfeeding

*Describe the nursing assessments of a newborn with Meconium Aspiration Syndrome.*

- Assess the amniotic fluid for meconium staining when the maternal membranes rupture. • Green-stained amniotic fluid suggests the presence of meconium in the amniotic fluid and should be reported immediately • After birth, note any yellowish-green staining of the umbilical cord, nails, & skin. This indicates that meconium has been present for some time. • barrel-shaped chest with an increased AP chest diameter (similar to COPD), prolonged tachypnea, progression from mild to severe respiratory distress, intercostal retractions, end-expiratory grunting, and cyanosis • coarse crackles and rhonchi • Chest x-rays show patchy, fluffy infiltrates unevenly distributed throughout the lungs and marked hyperaeration mixed with areas of atelectasis. • ABG analysis will indicate metabolic acidosis with a low blood pH, decreased PaO2, and increased PaCO2. - Direct visualization of the vocal cords for meconium staining using an appropriate size laryngoscope is needed to confirm the presence of meconium below the larynx. Take Note! *Standard prevention and treatment for meconium aspiration syndrome previously included suctioning the mouth and nares upon head delivery before body delivery. However, recent evidence suggests that aspiration occurs in utero, not at delivery; therefore, the infant's birth should not be impeded for suctioning. After full delivery, the infant should be handed to a neonatal team for evaluation and treatment. Although infants previously have been given intubation and airway suctioning, routine tracheal suction is recommended only for depressed infants (e.g., nonvigorous with depressed tone and respirations and/or heart rate <100 bpm) and those with respiratory symptoms. Use of orogastric suctioning to prevent MAS is not supported by evidence from current studies. Guidelines suggest not stimulating infants born with meconium staining with vigorous sucking, to avoid aspiration*

*Mild Preeclampsia*

- BP: >140/90 mm Hg after 20 weeks' gestation - Proteinuria: 300 mg/24 hr or greater than 1+ protein on a random dipstick urine sample - Seizures/coma: No - Hyperreflexia: No - Other signs and symptoms: Mild facial or hand edema Weight gain

*Know and be able to apply nursing implications and precautions when administering Magnesium Sulfate.* *nursing implications and precautions when administering Magnesium Sulfate*

- Blockage of neuromuscular transmission, vasodilation - Prevention and treatment of eclamptic seizures - Nursing implications: Loading dose of 4-6 g by IV in 100 mL of fluid administered over 15-20 minutes, followed by a maintenance dose of 2 g as a continuous IV infusion. Monitor serum magnesium levels closely. Assess DTRs and check for ankle clonus. *Have calcium gluconate readily available in case of toxicity.* *Monitor for signs and symptoms of toxicity: flushing, sweating, hypotension, and cardiac and central nervous system depression.*

*Nursing management for the client with hyperemesis gravidarum* focuses on promoting comfort by controlling the client's nausea and vomiting and promoting adequate nutrition. In addition, the nurse plays a major role in supporting and educating the client and her family.

- During the initial period, expect to withhold all oral food and fluids, maintaining NPO status to allow the gastrointestinal tract to rest. In addition, administer prescribed antiemetics to relieve the nausea and vomiting and IV fluids to replace fluid losses. Monitor the rate of infusion to prevent overload and assess the IV insertion site to prevent infiltration or infection. Also administer electrolyte replacement therapy as ordered to correct any imbalances, and periodically check serum electrolyte levels to evaluate the effectiveness of therapy. Provide physical comfort measures such as hygiene measures and oral care. Pay special attention to the environment, making sure to keep the area free of pungent odors. As the client's nausea and vomiting subside, gradually introduce oral fluids and foods in small amounts. Monitor intake and output and assess the client's tolerance to the increase in intake. - Women with hyperemesis gravidarum commonly are fatigued physically and emotionally. Many are exhausted, frustrated, and anxious. Offer reassurance that all interventions are directed toward promoting positive pregnancy outcomes for both the woman and her fetus. Providing information about the expected plan of care may help to alleviate the client's anxiety. Listen to her concerns and feelings, answering all questions honestly. Educate the woman and her family about the condition and its treatment options (Teaching Guidelines 19.1). Teach the client about therapeutic lifestyle changes, such as avoiding stressors and fatigue that may trigger nausea and vomiting. Offer ongoing support and encouragement and promote active participation in care decisions, thereby empowering the client and her family. Attempting to provide the client with a sense of control may help her overcome the feeling that she has lost control. If necessary, refer the client to a spiritual advisor or counseling. Also suggest possible local or national support groups that the client may contact for additional information. Arrange for possible home care follow-up for the client and reinforce discharge instructions to promote understanding. Timely counseling, balanced nutrition, pharmacotherapy, and emotional support are associated with favorable outcomes for the woman with this condition. Collaborate with community resources to ensure continuity of care.

*Explain the key factors in the physiological process of newborn cold stress* temperature regulation - key points

- Heat production is less than an adult. - Flexed position of the NB helps to prevent heat loss by diminishing the amount of body surface exposed to the environment. - Cold stress is a very real, serious complication which can affect the NB in adversely. - Cold stress is not a respecter of gestational age...it affects ALL metabolically and physiologically. - *Dry the newborn thoroughly after birth!* - *Drying the newborn immediately after birth using warmed blankets is essential to prevent heat loss through evaporation.* - Then the nurse would place a cap on the baby's head and wrap the newborn. - Assessing the newborn's temperature would occur once these measures were initiated to prevent heat loss. If baby starts to draw up and flex, it's probably starting to get cold

*Identify risk factors and nursing management associated with: o primary cesarean sections o repeat cesarean sections o Vaginal Birth After Cesarean sections Problems Who is a candidate Looking at strength of prior incision

1 in 3 women who gives birth in the United States today does so by a cesarean birth. The most common indications for primary cesarean births include, in order of frequency, labor dystocia, abnormal fetal heart rate (FHR) tracing, fetal malpresentation, multiple gestation, and suspected macrosomia. It is time to revisit the definition of labor dystocia because recent studies show that contemporary labor progresses at a rate substantially slower than what was historically taught. Labors today are often longer which may in part be due to higher body mass index (BMI), higher rates of labor induction, and the significant increase in the use of epidural anesthesia

*Gonorrhea* Bacterial not viral Tx: cephalosporin BFF: chlamydia Delivers vaginally and is positive? worried about eye stuff *gonorrhea s/s* [11] Between 50% and 90% of women infected are totally symptom-free. Because women are so frequently asymptomatic, they are regarded as a major factor in the spread of gonorrhea. If symptoms are present, they might include:

1. *Abnormal vaginal discharge* 2. *Dysuria* 3. *Cervicitis* 4. *Enlarged lymph glands locally* 5. *Abnormal vaginal bleeding* 6. *Bartholin abscess* 7. *PID* 8. *Neonatal conjunctivitis in newborns* 9. *Mild sore throat (for pharyngeal gonorrhea)* 10. *Rectal infection (itching, soreness, bleeding, discharge)* 11. *Perihepatitis* Assess the client's health history for risk factors, which may include low socioeconomic status, living in an urban area, single status, inconsistent use of barrier contraceptives, age <20, & multiple sex partners. Sometimes, a local gonorrhea infection is self-limiting (there is no further spread), but usually the organism ascends upward through the endocervical canal to the endometrium of the uterus, further on to the fallopian tubes, and out into the peritoneal cavity. When the peritoneum and the ovaries become involved, the condition is known as PID. The scarring to the fallopian tubes is permanent. This damage is a major cause of infertility and is a possible contributing factor in ectopic pregnancy. If gonorrhea remains untreated, it can enter the bloodstream and produce a disseminated gonococcal infection. *This severe form of infection can invade the joints (arthritis), the heart (endocarditis), the brain (meningitis), and the liver (toxic hepatitis).* The CDC recommends screening for all women at risk for gonorrhea. Pregnant women should be screened at the 1st prenatal visit and again at 36 weeks of gestation. Nucleic acid hybridization tests (GenProbe) are used for diagnosis. Any woman suspected of having gonorrhea should be tested for chlamydia also because coinfection (45%) is extremely common M: Chorioamnionitis, preterm birth, PROM, fetal growth restriction, postpartum sepsis F: Eye infection gonococcal ophthalmia which can cause blindness

BOX 2.4 STEPS TO DEVELOPING CULTURAL COMPETENCE [4] i'm not sure if this is included or not

1. *Cultural Self-Awareness*: Become aware of, appreciate, and become sensitive to the values, beliefs, customs, and behaviors that have shaped your own culture. Engage in self-exploration beyond your own culture and "see" clients from different cultures. Health care has a multicultural environment. Cultural diversity is awareness of the presence of differences among clients. Examine personal biases and prejudices toward other cultures. Become aware of differences in personal and clients' backgrounds. 2. *Cultural Knowledge*: Obtain knowledge about various worldviews of different cultures, such as through reading about different cultures, attending continuing education courses on different cultures, accessing websites, and attending cultural diversity conferences. Become familiar with culturally/ethnically diverse groups, worldviews, beliefs, practices, lifestyles, and problem-solving strategies. 3. *Cultural Skills*: Learn how to perform a competent cultural assessment. Assess each client's unique cultural values, beliefs, and practices without depending solely on written facts about specific cultural groups. Embracing ethics empowers mutual respect, equality, and trust. 4. *Cultural Encounter*: Engage in cross-cultural interactions with people from culturally diverse backgrounds, such as attending religious services or ceremonies and participating in important family events. Participate in as many cultural encounters as possible to avoid cultural stereotyping.

*s/s of magnesium toxicity*

1. *Decreased DTR's!!!* 2. Resp. depression 3. Oliguria 4. Decreased LOC 5. slurred speech 6. Nausea 7. Feeling of warmth 8. *Flushing (teach that flu symptoms are supposed to happen/common)* 9. Muscle weakness Antidote: *calcium gluconate* (always make sure it is available)

*Uterine contractions are monitored and assessed according to three parameters: frequency, duration, and intensity.*

1. Frequency refers to how often the contractions occur and is measured from the beginning of one contraction to the beginning of the next contraction. 2. Duration refers to how long a contraction lasts and is measured from the beginning of one contraction to the end of that same contraction. 3. *Intensity* refers to the strength of the contraction determined by manual palpation or measured by an internal intrauterine pressure catheter. The catheter is positioned in the uterine cavity through the cervix after the membranes have ruptured. It reports intensity by measuring the pressure of the amniotic fluid inside the uterus in millimeters of mercury. It is not recommended for routine use in low-risk laboring women due to the potential risk of infection and injury to the placenta or fetus. In a recent meta-analysis involving >2,000 laboring women, the researchers found an increase in surgical births and no advantages of using internal intrauterine pressure catheters over external monitoring during labor augmentation

*danger/warning signs of 1st trimester* [4]

1. Hyperemesis gravidarum: Severe, persistent vomiting 2. miscarriage: *Vaginal bleeding*, Cramping 3. ectopic pregnancy: Lower abdominal pain (usually on only 1 side), Dizziness, Pain radiating to shoulder 4. infection: Dysuria, Fever, diarrhea, & chills

*nursing responsibilities for working with an interpreter* BOX 2.1 TIPS FOR WORKING WITH AN INTERPRETER [15]

1. *Help interpreter prepare & understand what needs to be done ahead of time.* A few minutes of preparation may save a lot of time and help communication flow more smoothly in the long run. 2. *Introduce yourself to both parties.* It is the KEY to easy communication across the language barrier. 3. *Physically place yourselves so that you are facing your client, and your client is facing you!* Place the interpreter physically close to both of you, but *not in a position that breaks the line of sight between you and your client.* 4. *Begin with a min or 2 of light conversation to establish a rhythm before getting into the business at hand.* This establishes the quantity of speech in the conversation. 5. *Remember, the interpreter is the "communication bridge" and not the "content expert"!* The nurse's presence at teaching sessions is VITAL. 6. *Establish rapport with the interpreter.* 7. *Be patient! The interpreter's timing may not match that of others involved!* It often takes longer to say in some languages what has already been said in English; therefore, plan for more time than you normally would. 8. *Speak slowly and clearly! Avoid jargon! Use short sentences and be concise. Avoid interrupting the interpreter!* 9. *Pause every few sentences so the interpreter can translate your information!* After 30 seconds of speaking, stop and let the interpreter express the information. *Talk directly to the family/pt, NOT the interpreter!* 10. *Give the family and the interpreter a break.* Sessions that last longer than 20-30 min are too much for anyone's attention span and concentration. 11. *Express the information in 2-3 different ways if needed.* There may be cultural barriers as well as language and dialect differences that interfere with understanding. Interpreters may often know the correct communication protocols for the family. 12. *Use an interpreter to help ensure the family can read and understand translated written materials.* The interpreter can also help answer questions and evaluate learning. 13. *Avoid side conversations during sessions!* These can be uncomfortable for the family and jeopardize client-provider relationships and trust. 14. *Remember, just because someone speaks another language doesn't mean that he or she will make a good interpreter.* An interpreter who has no medical background may not understand or interpret correctly, no matter how good his or her language skills are. 15. *Do not use children as interpreters.* Doing so can affect family relationships, proper understanding, and compliance with health care issues.

Providing Postoperative Care R/T mastectomy For the woman who has had surgery to remove a malignant breast lump or an entire breast, excellent postoperative nursing care is crucial. Tell the woman what to expect in terms of symptoms and when they usually occur during treatment and after surgery. This allows women to anticipate these symptoms and proactively employ management strategies to improve their cancer experience. Postoperative care includes immediate postoperative care, pain management, care of the affected arm, wound care, mobility care, respiratory care, emotional care, referrals, and educational needs. [8]

1. *IMMEDIATE POSTOPERATIVE CARE* Assess the client's respiratory status by auscultating the lungs and observing the breathing pattern. Assess circulation; note vital signs, skin color, and skin temperature. Observe the client's neurologic status by evaluating the level of alertness and orientation. Monitor the wound for amount and color of drainage. Monitor the intravenous lines for patency, correct fluid, and rate. Assess the drainage tube for amount, color, and consistency of drainage. 2. *PAIN MANAGEMENT* Provide analgesics as needed. Reassure the woman that her pain will be controlled. Teach the woman how to communicate her pain intensity on a scale of 0 to 10, with 10 being the worst pain imaginable. Assess the client's pain level frequently and anticipate pain before assisting the woman to ambulate. 3. *AFFECTED ARM CARE* Elevate the affected arm on a pillow to promote lymph drainage. Make sure that no treatments are performed on the affected arm, including laboratory draws, intravenous lines, blood pressures, and so on. Place a sign above the bed to warn others not to touch the affected arm. 4. *WOUND CARE* Observe the wound often and empty drainage reservoirs as needed. Tell the client to report any evidence of infection early, such as fever, chills, or any area of redness or inflammation along the incision line. Also tell the client to report any increase in drainage, foul odor, or separation at the incision site. 5. *MOBILITY CARE* Perform active range-of-motion and arm exercises as ordered. Encourage self-care activities for successful rehabilitation. Perform dressing and drainage care; explain the care during the procedure. 6. *RESPIRATORY CARE* Assist with turning, coughing, and deep breathing every 2 hours. Explain that this helps to expand collapsed alveoli in the lungs, promotes faster clearance of inhalation agents from the body, and prevents postoperative pneumonia and atelectasis. 7. *EMOTIONAL CARE AND REFERRALS* Encourage the client to participate in her care. Assess her coping strategies preoperatively. Explain possible body image concerns after discharge. Promote the ACS web sites, which provide the latest cancer therapy news. Encourage the client to attend local support groups for breast cancer survivors, such as Reach to Recovery. 8. *EDUCATIONAL NEEDS* Provide follow-up information about adjunctive therapy. Explain that radiation therapy may start within weeks postoperatively. Discuss chemotherapy, its side effects and cycles, home care during treatment, and future monitoring strategies. Explain hormonal therapy, including antiestrogens or aromatase inhibitors. Teach progressive arm exercises to minimize lymphedema. Explain that ongoing surveillance is needed to detect recurrence of cancer or a new primary site and that the client will typically see the health care provider every 6 months.

AUB is similar to several other types of uterine bleeding disorders and sometimes overlaps these conditions. They include: [5] idk if this will be on the final or not

1. *Menorrhagia* (abnormally long, heavy periods, prolonged bleeding) 2. *Oligomenorrhea* (bleeding occurs at intervals of more than 35 days) 3. *Metrorrhagia* (bleeding between periods, irregular bleeding) 4. *Polymenorrhea* (too frequent periods) 5. Menometrorrhagia (excessive uterine bleeding at and between menstrual periods)

*Cold stress may result in what?! SATA TEST* [3]

1. *Respiratory distress* 2. *HYPOglycemia* 3. *jaundice* *can lead to the following problems if not reversed: depleted brown fat stores, increased oxygen needs, respiratory distress, increased glucose consumption leading to hypoglycemia, metabolic acidosis, jaundice, hypoxia, and decreased surfactant production!*

*Describe the respiratory assessment for a newborn and the signs and symptoms of respiratory distress in a preterm newborn.* *Before the newborn's lungs can maintain respiratory function, the following events must occur:*

1. *respiratory movement must be initiated* 2. *lungs must expand* 3. *functional residual capacity must be established* 4. *pulmonary blood flow must increase* 5. *cardiac output must be redistributed* *What stimulates the first big breath at birth?* *The babies cry reflex!* *the MOST critical and immediate adjustment a newborn makes at birth is the establishment of respirations!* - On NB assessment: *the nurse assesses a symmetry of expansion, clavicles/ribs; crackles normal first 30 min-1hr* - priority nursing diagnoses is Impaired Gas Exchange and Ineffective Breathing Pattern

*Postoperative nursing interventions for mastectomy* Tx pain T/C/DB Depending on side No sticks, BPs, etc in affected arm How to position arm *recommendations to prevent lymphedema* In conjunction with the mastectomy, lymph node surgery (removal of underarm nodes) may need to be done to reduce the risk of distant metastasis and improve a woman's chance of long-term survival. For women with a positive sentinel node biopsy, 10-20 underarm lymph nodes may need to be removed. Complications associated with axillary lymph node surgery include nerve damage during surgery, causing temporary numbness down the upper aspect of the arm; seroma formation (fluid build-up) followed by wound infection; restrictions in arm mobility (some women need physiotherapy); and lymphedema (swelling related to the lymph glands). In many women lymphedema can be avoided by: [4]

1. Avoiding using the affected arm for drawing blood, inserting intravenous lines, or measuring blood pressure (can cause trauma and possible infection) 2. Seeking medical care immediately if the affected arm swells 3. Wearing gloves when engaging in activities such as gardening that might cause injury 4. Wearing a well-fitted compression sleeve to promote drainage return

*Understand the classification, assessment findings, and management for the following types of HTN that can occur in pregnant women:* ["I love BP problems, be able to apply them to situations, differences, how to tx, s/s] o Gestational hypertension o Mild pre-eclampsia o Severe pre-eclampsia o Eclampsia o HELLP syndrome Look at notes for more stuff if needed The classification of hypertensive disorders in pregnancy currently consists of 5 categories:

1. Chronic hypertension: hypertension that exists prior to pregnancy or that develops before 20 weeks' gestation. 2. *Gestational hypertension*: blood pressure elevation (140/90 mm Hg) identified after 20 weeks' gestation without proteinuria. Blood pressure returns to normal by 12 weeks' postpartum. 3. *Preeclampsia*: most common hypertensive disorder of pregnancy, which develops with proteinuria after 20 weeks' gestation. It is a multisystem disease process, which is classified as mild or severe, depending on the severity of the organ dysfunction. 4. *Eclampsia*: Onset of seizure activity in a woman with preeclampsia. 5. Chronic hypertension with superimposed preeclampsia: occurs in approximately 20% of pregnant women with increased maternal and fetal morbidity rates

Assess the woman for *clinical manifestations of breast cancer*, such as changes in breast appearance and contour, which become apparent with advancing breast cancer. These changes include: [7]

1. Continued and persistent changes in the breast 2. A lump or thickening in one breast 3. Persistent nipple irritation 4. Unusual breast swelling or asymmetry 5. A lump or swelling in the axilla 6. Changes in skin color or texture (peau d'orange) 7. Nipple retraction, tenderness, or discharge

*Review the signs and symptoms FAS fetal Alcohol Syndrome Box 24.2* *BOX 24.2 CLINICAL PICTURE OF FETAL ALCOHOL SYNDROME* [22 & 3]

1. Microcephaly (head circumference <10th percentile) 2. Small palpebral (eyelid) fissures 3. Abnormally small eyes 4. Fetal growth restriction 5. Maxillary hypoplasia (flattened or absent) 6. Epicanthal folds (folds of skin of the upper eyelid over the eye) 7. Thin upper lip 8. Missing vertical groove in median portion of upper lip 9. Short upturned nose 10. Short birth length and low birthweight 11. Joint and limb defects 12. SGA 13. Altered palmar crease pattern 14. Prenatal or postnatal growth ≤10th% 15. Congenital cardiac defects (septal defects) 16. Delayed fine and gross motor development 17. Poor eye-hand coordination 18. Clinically significant brain abnormalities 19. Mental retardation 20. Narrow forehead 21. Performance substantially below expected level in cognitive or developmental functioning, executive or motor functioning, and attention or hyperactivity; social or language skills 22. Inadequate sucking reflex and poor appetite Diagnosis of fetal alcohol syndrome requires the presence of 3 findings: 1. Documentation of all 3 facial abnormalities 2. Documentation of growth deficits (height, weight, or both <10th%) 3. Documentation of CNS abnormalities (structural, neurologic, or functional)

*Know the teaching for an anemic pregnant woman taking an iron supplement.* Tidwell, Antepartum Complications *Teaching Guidelines 20.3 TEACHING FOR THE WOMAN WITH IRON-DEFICIENCY ANEMIA* [8]

1. Take your prenatal vitamin daily; if you miss a dose, take it as soon as you remember. 2. For best absorption, take iron supplements between meals. 3. Awareness of the side effects of iron supplementation 4. Avoid taking iron supplements with coffee, tea, chocolate, and high-fiber foods. 5. Eat foods rich in iron, such as: - Meats, green leafy vegetables, legumes, dried fruits, whole grains - Peanut butter, bean dip, whole-wheat fortified breads and cereals 6. For best iron absorption from foods, consume the food along with a food high in vitamin C. 7. Increase your exercise, fluids, and high-fiber foods to reduce constipation. 8. Plan frequent rest periods during the day.

BOX 1.5 KEY ELEMENTS OF INFORMED CONSENT

1. The decision maker must be of legal age in that state, with full civil rights, and must be competent (have the ability to make the decision). 2. Present information that is simple, concise, and appropriate to the level of education and language of the individual responsible for making the decision. 3. The decision must be voluntary, and without coercion, force, or influence of duress. 4. Have a witness to the process of informed consent. 5. Have the witness sign the consent form.

the 2 biochemical markers for PTL

1. fetal fibronectins 2. salivary estriol

*danger/warning signs of 2nd & 3rd trimester* [8]

1. preterm labor: Uterine contractions, Pelvic pressure, Cramping 2. *premature ROM: Sudden gush or leaking of fluid* 3. DVT: Pain in calf, exacerbated with foot flexion 4. fetal jeopardy: Absence of fetal movement > 12hr, Decrease in fetal movement >24hr, need to see 10 movements in 2 hrs 5. preeclampsia: Epigastric or abdominal pain, Visual disturbances (blurring, double, spots), HAs, Edema of face & hands, Muscular irritability, convulsion 6. placental previa: Painless vaginal bleeding 7. placental abruption: Severe abdominal pain, Vaginal bleeding 8. kidney stone/infection: severe back pain, Flank pain

*CHART 3-4 American Nurses Association Code of Ethics for Nurses*

1. the nurse, in all professional relationships, practices with *compassion and respect for the inherent dignity, worth, and uniqueness of every individual, unrestricted by considerations of social or economic status, personal attributes, or the nature of health problems* 2. The nurse's primary commitment is to the *patient*, whether an individual, family, group, or community. 3. The nurse promotes, advocates for, and strives to *protect* the health, safety, and rights of the patient. 4. The nurse is responsible and accountable for individual nursing practice and determines the appropriate *delegation* of tasks consistent with the nurse's obligation to provide optimum patient care. 5. The nurse *owes the same duties to self as to others*, including the responsibility to preserve integrity and safety, to maintain competence, and to continue personal and professional growth. 6. The nurse participates in establishing, maintaining, and improving *health care environments* and conditions of employment conducive to the provision of quality health care and consistent with the values of the profession through individual and collective action. 7. The nurse participates in the *advancement* of the profession through contributions to practice, education, administration, and knowledge development. 8. The nurse *collaborates* with other health professionals and the public in promoting community, national, and international efforts to meet health needs. 9. The profession of nursing, as represented by associations and their members, is responsible for *articulating nursing values, for maintaining the integrity of the profession and its practice, and for shaping social policy*

*Discuss the nursing care of a patient with an episiotomy.* o Immediate L &D Ice packs o Later, like next day Sitz baths, etc.

3rd stage: Applying an ice pack to the perineal area to provide comfort to episiotomy if indicated 4th stage: Demonstrating the use of the portable sitz bath as a comfort measure for her perineum if she had a laceration or an episiotomy repair An episiotomy is an incision made in the perineum to enlarge the vaginal outlet and theoretically to shorten the second stage of labor. Alternative measures such as warm compresses and continual massage with oil have been successful in stretching the perineal area to prevent cutting it. Certified nurse midwives can cut and repair episiotomies, but they frequently use alternative measures if possible. - The midline episiotomy has been the most commonly used one in the United States because it can be easily repaired and causes the least amount of pain. The application of warmed compresses and/or intrapartum perineal massage is associated with a decrease in trauma to the perineal area and reduced the need for an episiotomy. Routine episiotomy has declined since liberal usage has been discouraged by ACOG, except to avoid several maternal lacerations or to expedite difficult births. Anal sphincter laceration rates with spontaneous vaginal delivery have decreased, likely reflecting the decreased usage of episiotomy. The decline in operative vaginal delivery corresponds with a sharp increase in cesarean births, which may indicate that health care providers are favoring cesarean births for difficult births. Take Note! *Restrictive use of episiotomy has been recommended by ACOG given the risks of the procedure and unclear benefits of routine use* The vagina and perineal areas are quite stretched and edematous following a vaginal birth. Assess the perineum, including the episiotomy if present, for possible hematoma formation. Suspect a hematoma if the woman reports excruciating pain or cannot void or if a mass is noted in the perineal area. Also assess for hemorrhoids, which can cause discomfort.

*Severe Preeclampsia*

>160/110 mm Hg >500 mg/24 hr; greater than 3+ on random dipstick urine sample - Seizures/coma: No - Hyperreflexia: No - Other s/s: Headache Oliguria Blurred vision, scotomata (blind spots) Pulmonary edema Thrombocytopenia (platelet count <100,000 platelets/mm3) Cerebral disturbances Epigastric or RUQ pain HELLP

*Eclampsia*

>160/110 mm Hg Marked proteinuria - Seizures/coma: Yes - Hyperreflexia: Yes - other s/s: Severe headache Generalized edema RUQ or epigastric pain Visual disturbances Cerebral hemorrhage Renal failure HELLP

*tocolytics - terbutaline (Brethine)*

Action: relaxes smooth muscle, inhibiting uterine activity Dosage - Subq: 0.25mg q 20-30 min up to 3hrs PO: 2.5-5mg q 4-6 hours S/E: Cardiac, pulmonary, vascular Nursing Interventions Side Effects: tachycardia, palpitations, hypotension, SOB, nervousness, restlessness, HA, tremor, insomnia, angina, hyperglycemia Nursing interventions: One to one care, VS prior to each administration, hold dose and notify MD if pulse exceeds 120bpm, assess lung sounds for dyspnea and crackles indicating possible development of pulmonary edema Monitor lytes and glucose levels Most common Watch mom's HR Give SQ in back of arm Will feel like you drank too much coffee, SOB, fidgety Teach if they are having that feeling before 2nd dose- that's a good sign Check VS, monitor before 2nd dose *Cant give 2nd dose if HR>120* PO: Teach mom 2 set alarm to take it every 6 hours (or whoever prescribed)!!! Can't just take it when you feel like it

glucocorticoids - betamethasone (Celestone)

Action: stimulates & accelerates fetal lung maturation Prevents or reduces the severity of resp. distress syndrome in preterm infants between 24-34 wks gestation. Dosing: 12 mg IM x 2, 24 hrs apart Need to monitor mother's glucose level Also associated with decreased incidence of intraventricular hemorrhage and necrotizing enterocolitis Given to stimulate fetal surfactant production to accelerate fetal lung maturity Steroids Want to get at least 2 doses in pt IM Give at least 24 hours apart But can give 12 hours apart if we think mom isn't going to last Doesn't work on stopping pre-term labor, it's an additive to help the baby

*providing follow-up care for preeclampsia*

After delivery of the newborn, continue to monitor the client for signs and symptoms of preeclampsia/eclampsia for at least 48 hours. Expect to continue to administer magnesium sulfate infusion for 24 hours to prevent seizure activity, and monitor serum magnesium levels for toxicity. Assess vital signs at least every 4 hours, along with routine postpartum assessments: fundus, lochia, breasts, bladder, bowels, and the woman's emotional state. Monitor urine output closely. Diuresis is a positive sign that, along with a decrease in proteinuria, signals resolution of the disease.

*Ultrasound (US)* look at notes to see if there is extra stuff

A gestational or birth calculator or wheel can also be used to calculate the due date, some practitioners use US to more accurately determine the gestational age and date the pregnancy, *US is typically the most accurate method of dating a pregnancy* - Since its introduction in the late 1950s, ultrasonography has become a very useful diagnostic tool in obstetrics. Real-time scanners can produce a continuous picture of the fetus on a monitor screen. A transducer that emits high-frequency sound waves is placed on the mother's abdomen and moved to visualize the fetus. The fetal heartbeat and any malformations in the fetus can be assessed and measurements can be made accurately from the picture on the monitor screen. - Obstetric ultrasound is a standard component of prenatal care used to identify pregnancy complications and to establish an accurate gestational age in order to improve pregnancy outcomes. Because the ultrasound procedure is noninvasive, it is a safe, but not evidence-based practice for low-risk women, accurate, and cost-effective tool. It provides important information about fetal activity, growth, and gestational age; assesses fetal well-being; and determines the need for invasive intrauterine tests. - There are no hard-and-fast rules as to the number of ultrasounds a woman should have during her pregnancy. A low-risk woman does not necessarily require any, but most practices do them as part of their prenatal care routine. A transvaginal ultrasound may be performed in the first trimester to confirm pregnancy, exclude ectopic (in which a fertilized egg implants somewhere other than the main cavity of the uterus) or molar (hydatidiform mole, a benign tumor that develops in the uterus) pregnancies, and confirm cardiac pulsation. A second abdominal scan may be performed at about 18 to 20 weeks to look for congenital malformations, exclude multifetal pregnancies, and verify dates and growth. A third abdominal scan may be done at around 34 weeks to evaluate fetal size, assess fetal growth, and verify placental position. An ultrasound is used to confirm placental location during amniocentesis and to provide visualization during chorionic villus sampling (CVS). An ultrasound is also ordered whenever an abnormality is suspected. - During the past several years, ultrasound technology has advanced significantly. Now available for expecting parents is 3D/4D ultrasound imaging. Unlike traditional 2D imaging, which takes a look at the developing fetus from one angle (thus creating the "flat" image), 3D imaging takes a view of the fetus from three different angles. Software then takes these three images and merges them to produce a three-dimensional image. Because the fourth dimension is time and movement, with 4D parents are able to watch the live movements of their fetus in 3D. - Nursing management during the ultrasound procedure focuses on educating the woman about the ultrasound test and reassuring her that she will not experience any sensation from the sound waves during the test. No special client preparation is needed before performing the ultrasound, although in early pregnancy the woman may need to have a full bladder. Inform her that she may experience some discomfort from the pressure on the full bladder during the scan, but it will last only a short time. Tell the client that the conducting gel used on the abdomen during the scan may feel cold initially.

*mgmt of gestational HTN*

A recent study found that progesterone supplementation during the first trimester significantly reduced the incidence of gestational hypertension and fetal distress in primigravida women. This supplementation might be a future therapy with addition studies to validate it

tocolytics - Magnesium Sulfate

Action: relaxes smooth muscle Dosage: initial bolus of 4-6g over 15-30 min Maintenance dose: 1-4 g/hr until contractions stop or decrease to one contraction or less in 10-15 min Used in HTN as well as a tocolytic Adverse reactions: Hot flushes, seating, N/V, drowsiness, lethargy, SOB, decreased or diminished DTR, decrease urinary output, decrease blood pressure Nursing Interventions: Assess for side effects, monitor mag levels, assess vs, monitor fhr and UC, have *calcium gluconate* ready (antidote) Know that *therapeutic level in 4-7 and becomes toxic above 7!!!* used in preeclampsia, eclampsia to relax smooth muscle (uterus) People on it will have a long labor bc its fighting w/ Pitocin Stops preterm labor 4-6 labor at 1st Maintenance 1-4 Want level if not respiratory goes down, reflexes goes down

*Review the process to administer an injection of vitamin K to a newborn* (Drug Guide 18.1) Phytonadione (vitamin K [Aqua-MEPHYTON, Konakion, Mephyton]) [6]

Action/Indication: 1. Provides the newborn with vitamin K (necessary for production of adequate clotting factors II, VII, IX, and X by the liver) during the first week of birth until newborn can manufacture it 2. Prevents vitamin K deficiency bleeding (VKDB) of the newborn *Nursing Implications:* 1. Administer w/i 1-2 hr after birth 2. Give as an IM injection at a 90-degree angle into the outer middle third of the vastus lateralis muscle. 3. Use a 25-gauge, 5/8-in needle for injection. 4. Hold the leg firmly and inject medication slowly. 5. Adhere to standard precautions. 6. Assess for bleeding at injection site after administration.

tocolytics - nifedipine (Procardia)

Action: relaxes smooth muscle Dosage: PO: Initially 30mg then 10-20mg q 4-8 hrs Adverse reactions: dizziness, HA, nervousness, palpitations, peripheral edema, hypotension Nursing Interventions: do not use with Mag, assess BP and pulse before administration (vasodilation), monitor I&O, do not crush, open, break or chew tabs Smooth muscle relaxant Watch BP bc it will tank on you Baby can't get correct perfusion, blood, oxygen causing babies HR to go down

tocolytics - indomethacin

Action: relaxes smooth muscle Dosage: Rectally: 50mg PO: 50-100mg initially then 25-50mg q4-6 hrs up to 48 hrs Adverse reactions: N/V, dyspepsia (a vague discomfort in the upper abdomen or chest that may be described as gas, a feeling of fullness, gnawing, or burning), dizziness, can cause a *premature closure of ductus arteriosus* in fetus, oligohydramnios Maternal adverse reaction of increase bleeding due to decreased platelet aggregation Nursing interventions 1. Do not use with women with any potential for bleeding 2. US assessment for fetal compromise, decreased amniotic fluid, ductus arteriosus 3. Monitor for pph 4. Used for gestational age less than 32 weeks 5. Contraindications: bleeding potential, peptic ulcer disease, oligohydramnios Ductus arterious Have to be <32 weeks to give it to them HIGH RISK

fetal fibronectins

Appearance between 24 and 34 weeks gestation predicts labor A negative fetal fibronectin test is reliable to 94% that preterm labor will not occur Fetal fibronectin: glue that helps attach the membranes to the lower part of the uterus Shortly before labor, lower part of the uterus begins to change shape Causes membranes to separate from the wall Fibronectin is released and mingles with cervical and vaginal secretions fFN test measures the levels in secretions Appear in cervical canal early in pregnancy and then again in late pregnancy Done by vaginal swab Cannot have had a vaginal exam done within 24 hours of swab For when you're still not sure if shes in preterm labor It's the glue that holds the amniotic membrane to the uterine wall We check by doing a vaginal slough and put it on a slide or send it down to the lab Look to see if there is febal fibronectins in that swab If there is pt is going into preterm labor 90% sure

*clinical manifestations of placenta previa*

Ask the client if she has any problems associated with bleeding, now or in the recent past. The classical clinical presentation is painless, bright-red vaginal bleeding occurring during the second or third trimester. The initial bleeding usually is not profuse and it ceases spontaneously, only to recur again. The first episode of bleeding occurs (on average) at 27 to 32 weeks' gestation. The bleeding is thought to arise secondary to the thinning of the lower uterine segment in preparation for the onset of labor. When the bleeding occurs at the implantation site in the lower uterus, the uterus cannot contract adequately and stop the flow of blood from the open vessels. Typically with normal placental implantation in the upper uterus, minor disruptive placental attachment is not a problem, because there is a larger volume of myometrial tissue able to contract and constrict bleeding vessels. Assess the client for uterine contractions, which may or may not occur with the bleeding. Palpate the uterus; typically it is soft and nontender on examination. Auscultate the fetal heart rate; it commonly is within normal parameters. Fetal distress is usually absent but may occur when cord problems arise, such as umbilical cord prolapse or cord compression, or when the client has experienced blood loss to the extent that maternal shock or placental abruption has occurred - To validate the position of the placenta, a transvaginal ultrasound is done. In addition, magnetic resonance imaging (MRI) may be ordered when preparing for delivery because it allows identification of placenta accreta (placenta abnormally adherent to the myometrium), increta (placenta accreta with penetration of the myometrium), or percreta (placenta accreta with invasion of the myometrium to the peritoneal covering, causing rupture of the uterus) in addition to placenta previa. These placental abnormalities, although rare, carry a very high morbidity and mortality rate, possibly necessitating a hysterectomy at delivery.

*nursing assessment ABO & Rh*

At the 1st prenatal visit, determine the woman's blood type and Rh status. Also obtain a thorough health history, noting any reports of previous events involving hemorrhage to delineate the risk for prior sensitization. When the client's history reveals an Rh-negative mother who may be pregnant with an Rh-positive fetus, prepare the client for an antibody screen (indirect Coombs test) to determine whether she has developed isoimmunity to the Rh antigen. This test detects unexpected circulating antibodies in a woman's serum that could be harmful to the fetus

using an interpreter (from the book)

Attempting to communicate with a family who does not speak English can be one of the most frustrating situations in which health care providers find themselves. - In this situation, trained interpreters are an invaluable aid and an essential component of client and family education. - Whether working with an interpreter in person or over the phone, it is important to coordinate efforts so that both the family and the interpreter understand the information to be communicated. - Working as a team, the nurse questions or informs and the interpreter conveys the information completely and accurately. - Well-trained translators can also help prevent cultural missteps and can help guide the health care provider to provide information in a more culturally accepted manner

*Teaching Guidelines 19.1 TEACHING TO MINIMIZE NAUSEA AND VOMITING*

Avoid noxious stimuli - such as strong flavors, perfumes, or strong odors such as frying bacon - that might trigger nausea and vomiting. Avoid tight waistbands to minimize pressure on abdomen. Eat small, frequent meals throughout the day—six small meals. Separate fluids from solids by consuming fluids in between meals. Avoid lying down or reclining for at least 2 hours after eating. Use high-protein supplement drinks. Avoid foods high in fat. Increase your intake of carbonated beverages. Increase your exposure to fresh air to improve symptoms. Eat when you are hungry, regardless of normal mealtimes. Drink herbal teas containing peppermint or ginger. Avoid fatigue and learn how to manage stress in life. Schedule daily rest periods to avoid becoming overtired. Eat foods that settle the stomach, such as dry crackers, toast, or soda.

BURP magnesium toxicity

BP decreased Urine output decreased RR <12 Patella reflex absent

*Infants of Diabetic Mothers*

Be alert for hypoglycemia, which may occur immediately after birth or within an hour. Assess blood glucose levels, which should remain above 40 mg/dL *Closely assess the newborn for signs of hypoglycemia including:* listlessness hypotonia apathy poor feeding apneic episodes with a drop in oxygen saturation cyanosis temperature instability pallor and sweating tremors irritability seizures

FIGURE 24.6. Neonatal abstinence scoring system.

Central Nervous System Disturbances: Excessive high-pitched cry Continuous high-pitched cry Sleeps <1 hr after feeding Sleeps <2 hrs after feeding Sleeps <3 hrs after feeding Hyperactive Moro reflex Markedly hyperactive Moro reflex Mild tremors disturbed Moderate-severe tremors disturbed Mild tremors undisturbed Moderate-severe tremors undisturbed Increased muscle tone Excoloration (specify area) Myoclonic Jerks Generalized convulsions Metabolic/Vasomotor/Resp. Disturbances: Sweating Fever <101 (99-100.8/37.2-38.2) Fever >101 (38.2 and >) Frequent yawning (>3-4 times/interval) Mottling Nasal stuffiness Sneezing (>3-4 times/interval) Nasal flaring RR > 60 RR > 60 with retractions GI Disturbances: Excessive sucking Poor feeding Regurgitation Projectile vomiting Loose stools Watery stools Total Score:

TABLE 14.1 INTERPRETING FHR PATTERNS

Category I: normal Predictive of normal fetal acid-base status and do not require intervention • Baseline rate (110-160 bpm) • Baseline variability moderate • Present or absent accelerations • Present or absent early decelerations • No late or variable decelerations Can be monitored with intermittent auscultation during labor Category II: indeterminate Not predictive of abnormal fetal acid-base status, but require evaluation and continued surveillance • Fetal tachycardia (>160 bpm) present • Bradycardia (<110 bpm) not accompanied by absent baseline variability • Absent baseline variability not accompanied by recurrent decelerations • Minimal or marked variability • Recurrent late decelerations with moderate baseline variability • Recurrent variable decelerations accompanied by minimal or moderate baseline variability, overshoots, or shoulders • Prolonged decelerations >2 min but <10 min Category III: abnormal Predictive of abnormal fetus acid-base status and require intervention • Fetal bradycardia (<110 bpm) • Recurrent late decelerations • Recurrent variable decelerations—declining or absent • Sinusoidal pattern (smooth, undulating baseline)

PROVIDING PREOPERATIVE CARE R/T Cesarean Sections

Client preparation varies depending on whether the cesarean birth is planned or unplanned. The major difference is the time allotted for preparation and teaching. In an unplanned cesarean birth, institute measures quickly to ensure the best outcomes for the mother and fetus. Ensure that the woman has signed an informed consent, and allow for discussion of fears and expectations. Provide essential teaching and explanations to reduce the woman's fears and anxieties. Ascertain the client's and family's understanding of the surgical procedure. Reinforce the reasons for surgery given by the surgeon. Outline the procedure and expectations of the surgical experience. Ensure that all diagnostic tests ordered have been completed, and evaluate the results. Explain to the woman and her family about what to expect postoperatively. Reassure the woman that pain management will be provided throughout the procedure and afterward. Encourage the woman to report any pain. Ask the woman about the time she last had anything to eat or drink. Document the time and what was consumed. Throughout the preparations, assess maternal and fetal status frequently. Provide preoperative teaching to reduce the risk of postoperative complications. Demonstrate the use of the incentive spirometer and deep-breathing and leg exercises. Instruct the woman on how to splint her incision. Complete the preoperative procedures, which may include: 1. Preparing the surgical site as ordered 2. Starting an intravenous infusion for fluid replacement therapy as ordered 3. Inserting an indwelling (Foley) catheter and informing the client about how long it will remain in place (usually 24 hours) 4. Administering any preoperative medications as ordered; documenting the time administered and the client's reaction Maintain a calm, confident manner in all interactions with the client and family. Help transport the client and her partner to the operative area.

bed rest for PTL

Common treatment for preterm labor Cessation of all but most basic needs Risks of bed rest: Decreased muscle tone Weight loss, calcium loss Glucose intolerance Calcium loss can lead to bone demineralization Constipation Fatigue, isolation, anxiety & depression Bedrest helps to decrease potential for over exertion Requires a good support system Need activities to prevent boredom Need help with housework and children Support groups Home monitoring twice daily Least invasive measure 1st: bed rest A lot of studies say it doesn't do a lot But if you tell a pt there is no restrictions they won't chill out Problems w/ it: decreased muscle tone (more weight bc of baby), release of Calcium (which we need so bad), overeating from boredom, constipation (which preg. Women are already at risk for), fatigue, isolation, depression, anxiety, fidgeting

*mgmt for mild preeclampsia*

Conservative strategies for mild preeclampsia are used if the woman exhibits no signs of renal or hepatic dysfunction or coagulopathy. A woman with mild elevations in blood pressure may be placed on bed rest at home. She is encouraged to rest as much as possible in the lateral recumbent position to improve uteroplacental blood flow, reduce her blood pressure, and promote diuresis. In addition, antepartal visits and diagnostic testing - such as CBC, clotting studies, liver enzymes, and platelet levels - increase in frequency. The woman will be asked to monitor her blood pressure daily (every 4 to 6 hours while awake) and report any increased readings; she will also measure the amount of protein found in urine using a dipstick and will weigh herself for any weight gain. She also should take daily fetal movement counts, and if there is any decrease in movement, she needs to be evaluated by her health care provider that day. A balanced, nutritional diet with no sodium restriction is advised. In addition, she is encouraged to drink six to eight 8-oz glasses of water daily. If home management fails to reduce the blood pressure, admission to the hospital is warranted and the treatment strategy is individualized based on the severity of the condition and the gestational age at the time of diagnosis. During the hospitalization, the woman with mild preeclampsia is monitored closely for signs and symptoms of severe preeclampsia or impending eclampsia (e.g., persistent headache, hyperreflexia). Blood pressure measurements are frequently recorded along with daily weights to detect excessive weight gain resulting from edema. Fetal surveillance is instituted in the form of daily fetal movement counts, nonstress testing, and serial ultrasounds to evaluate fetal growth and amniotic fluid volume to confirm fetal well-being. Expectant management (watchful waiting) usually continues until the pregnancy reaches term, fetal lung maturity is documented, or complications develop that warrant immediate birth. Women with mild preeclampsia are at greatest risk for postpartum hypertension. Prevention of disease progression is the focus of treatment during labor. Blood pressure is monitored frequently and a quiet environment is important to minimize the risk of stimulation and to promote rest. IV magnesium sulfate is infused to prevent any seizure activity, along with antihypertensives if blood pressure values begin to rise. Calcium gluconate is kept at the bedside in case the magnesium level becomes toxic. Continued close monitoring of neurologic status is warranted to detect any signs or symptoms of *hypoxemia*, impending seizure activity, or increased intracranial pressure. An indwelling urinary (Foley) catheter usually is inserted to allow for accurate measurement of urine output.

PTL early recognition & diagnosis 3 major diagnostic criteria

Essential to implement interventions successfully such as tocolytic therapy and administration of antenatal glucocorticoids 1. *Gestational age* between 20-37 weeks 2. *Uterine activity (contractions)*, increase even w/ laying down and increasing fluids 3. *Progressive cervical change*: Cervical effacement of 80% or cervical dilation of 2 cm or greater Put them on the monitor - look for contractions, Check them- exposed, thick, babies high, closed Water not broken so doing alright and just need fluid - all good

*preeclampsia*

Development of hypertension with proteinuria or edema, or both, due to pregnancy or the influence of a recent pregnancy; it usually occurs after the 20th week of gestation, but may develop before this time in the presence of trophoblastic disease. - The "cure" for preeclampsia/eclampsia is always delivery of the placenta. The resolution following expulsion of the placenta supports theories related to the placental influence on the disease - According to recent studies, prevention of preeclampsia should be considered with daily low-dose aspirin from 12 weeks' gestation and onward to women identified at high risk for it. - While women with chronic hypertension or a personal history of preeclampsia should receive aspirin during pregnancy, further research should be ongoing to predict preeclampsia in low-risk women

*gestational hypertension*

Hypertension during pregnancy in a previously normotensive woman or aggravation of hypertension during pregnancy in a hypertensive woman. SYN pregnancy-induced hypertension category is used in women with nonproteinuric hypertension of pregnancy, in which the pathophysiologic disturbances of the preeclampsia syndrome do not develop before giving birth. Gestational hypertension is a temporary diagnosis for hypertensive pregnant women who do not meet the criteria for preeclampsia (both hypertension and possibly proteinuria) or chronic hypertension (hypertension first detected before the 20th week of pregnancy). - Previously, was known as pregnancy-induced hypertension or toxemia of pregnancy, but these terms are no longer used - can be differentiated from chronic hypertension, which appears before the 20th week of gestation; or hypertension before the current pregnancy, which continues after the woman gives birth.

*understand what situations are occurring that fall under the topic of ethical issues/dilemmas in women's health*

Ethical issues are increasing as technology increases and includes: 1. General issues of health and wellness of women 2. Genetic and embryo research 3. Conception and reproduction 4. Pregnancy and maternal/fetal issues 5. Neonate health and wellness 6. Contraception and Abortion - until fairly recently, the fetus was viewed legally as a non-person. Mother and fetus were viewed as one complex client - the pregnant woman - of which the fetus was an essential part. - These interventions infringe on the autonomy of the mother. Attempts have been made to criminalize the behavior of women who fail to follow a physician's advice or who engage in risky behaviors that are considered harmful to the fetus. - In 2005, Arkansas passed a bill to include in the definition of neglect the causing of a newborn child to be born with an illegal substance in his or her system or born with a health problem as a result of the pregnant mother's use before birth of an illegal substance. The mother is then reported to the state for possible child abuse. Only HCP are acceptable to make these reports. Other states have similar laws, so you need to know the law in the state in which you practice [Garrets Law!!!] - Refuses interventions on behalf of the fetus Forced intervention may include: 1. Forced cesarean birth 2. Coercion of mothers who practice high-risk behaviors (substance abuse) 3. Mandating experimental in-utero therapy or surgery to correct birth defects 4. Intrauterine fetal surgery, Fetoscopy 5. Therapeutic insemination 6. Genetic engineering, Stem cell research 7. Surgery for infertility 8. "Test tube" babies 9. Treatment of very-low-birth-weight (VLBW) babies 10. Surrogate motherhood 11. Sex selection 12. Selective reduction of multiple pregnancies 13. Termination of pregnancy following prenatal diagnosis 14. Cloning - Several areas are of particular importance to the health care of women and children. These include abortion, substance abuse, fetal therapy, maternal-fetal conflict, stem cell research, umbilical cord blood banking, informed consent, client rights, and confidentiality.

Dilation is dependent on the pressure of the presenting part and the contraction and retraction of the uterus. The diameter of the cervical os increases from less than 1 cm to approximately 10 cm to allow for birth. When the cervix is fully dilated, it is no longer palpable on vaginal examination. Descriptions may include the following:

External cervical os closed: 0 cm dilated External cervical os half open: 5 cm dilated External cervical os fully open: 10 cm dilated During early labor, uterine contractions are described as mild, they last about 30 seconds, and they occur about every 5 to 7 minutes. As labor progresses, contractions last longer (60 seconds), occur more frequently (2 to 3 minutes apart), and are described as being moderate to high in intensity. Each contraction has 3 phases: increment (buildup of the contraction), acme (peak or highest intensity), and decrement (descent or relaxation of the uterine muscle fibers; Fig. 13.13).

*Concepts of family-centered care* *concepts of Family Systems Theory*

Families are more *circular* than linear! Recognizing *changes in 1 member affect the entire family!!!* Family members influence and interact with each other View family as a *unit*, not in individual pieces Each member is unique and takes on several roles Take Note!: *Nurses can help to shape a person's lifelong perceptions of health and health services. An understanding of how the woman's, child's, and family's culture affects their health practices gives the nurse an opportunity to incorporate appropriate and beneficial health practices into the family's cultural milieu, providing sources of strength rather than areas of conflict.* Take Note! *NEVER make assumptions about a family's religious or spiritual affiliation. Although they may belong to a particular religion, family members may not adhere to all of its beliefs or participate in all aspects of the religion. Be alert for clues that would provide insight into their specific beliefs.*

salivary estriol

Form of estrogen produced by the fetus Present in plasma at 9 wks Levels increase before preterm birth Specimens are collected by the woman at home every 2 weeks for approx 10 weeks High predictive value Estriol is a form of estrogen produced by the placenta About 3-5 weeks before labor begins the amount of estriol in placenta increases dramatically Estriol enters the mother's blood and other body fluids (salvia) By measuring level can help predict when labor might begin Another way to check if in preterm labor Its released early and late w/I preg in mom Take a cheek swab to see if there is a presence of estriol If there is, most likely she's going into preterm labor

*Nursing Management ABO & Rh*

If the indirect Coombs test is negative (meaning no antibodies are present), then the woman is a candidate for RhoGAM. If the test is positive, RhoGAM is of no value because isoimmunization has occurred. In this case, the fetus is carefully monitored for hemolytic disease. The incidence of isoimmunization has declined dramatically as a result of prenatal and postnatal RhoGAM administration after any event in which blood transfer may occur. The standard dose is 300 mcg, which is effective for 15 mL of fetal blood cells. Rh immunoglobulin helps to destroy any fetal cells in the maternal circulation before sensitization occurs, thus inhibiting maternal antibody production. This provides temporary passive immunity, thereby preventing maternal sensitization. The current recommendation is for every Rh-negative nonimmunized woman to receive RhoGAM at some point between 28 and 32 weeks' gestation and again within 72 hours after giving birth. Other indications for RhoGAM include: Ectopic pregnancy Chorionic villus sampling Amniocentesis Prenatal hemorrhage Molar pregnancy Maternal trauma Percutaneous umbilical sampling Therapeutic or spontaneous abortion Fetal death Fetal surgery Despite the availability of RhoGAM and laboratory tests to identify women and newborns at risk, isoimmunization remains a serious clinical reality that continues to contribute to perinatal and neonatal mortality. Nurses, as client advocates, are in a unique position to make sure test results are brought to the health care provider's attention so appropriate interventions can be initiated. In addition, nurses must stay abreast of current literature and research regarding isoimmunization and its management. Stress to all women that early prenatal care can help identify and prevent this condition. Because Rh incompatibility is preventable with the use of RhoGAM, prevention remains the best treatment. Nurses can make a tremendous impact to ensure positive outcomes for the greatest possible number of pregnancies through education.

Special Situations to Informed Consent

If the parent is not available, then the person in charge (relative, babysitter, or teacher) may give consent for emergency treatment if that person has a signed form from the parent or legal guardian allowing him or her to do so - During an emergency situation, a verbal consent via the telephone may be obtained - Two witnesses must be listening simultaneously and will sign the consent form, indicating that consent was received via telephone - Health care providers can provide emergency treatment to a child without consent if they have made reasonable attempts to contact the child's parent or legal guardian - In urgent or emergent situations, appropriate medical care never should be delayed or withheld due to an inability to obtain consent - *Certain federal laws, such as the Emergency Medical Treatment and Labor Act (EMTALA), require that every client who presents at an ER is given a medical examination regardless of informed consent or reimbursement ability*

*Insulin for Management of Gestational Diabetes* (from book)

Insulin remains the medication of choice for glycemic control if oral medications, diet, or exercise fail to yield results in pregnant and lactating women with any type of diabetes. • Generally, insulin doses are reduced in the 1st trimester to prevent hypoglycemia resulting from increased insulin sensitivity as well as from n/v • Newer short-acting insulins such as lispro (Humalog) and aspart (NovoLog), which do NOT cross the placenta, may help reduce postprandial hyperglycemia, episodes of hypoglycemia b/w meals • Target fasting glucose values of 60 to 90 mg/dL and 1-hour postprandial values < 120 mg/dL are necessary to provide good glycemic control and good pregnancy outcomes • Changes in diet and activity level add to the need for changes in insulin dosages throughout pregnancy. • Insulin regimens vary, and controversy remains over the best strategy for insulin delivery in pregnancy. • Many HCPs use a split-dose therapy (2/3 of daily dose AM and remaining 1/3 PM). • Others advocate the use of an insulin pump to deliver a continuous subcutaneous insulin infusion. • Regardless of which protocol is used, frequent blood glucose measurements are necessary, and the insulin dosage is adjusted on the basis of daily glucose levels • Insulin therapy or oral hypoglycemic agents, in addition to diet and exercise, are main elements of achieving glycemic control. • Close maternal and fetal surveillance is also essential • Frequent laboratory tests are done during pregnancy to monitor the woman's status and glucose control. • Fetal surveillance via diagnostic testing aids in evaluating fetal well-being and assisting in determining the best time for birth.

*Nursing considerations when communicating with an interpreter* *characteristics and responsibilities of a good medical interpreter* [4]

Interpreters should ideally have: 1. The *same native language, religion & country of origin* 2. *Health-related language skills and experience* 3. *Maturity to keep confidentiality* 4. *Same sex as patient* Best choice: a trained medical interpreter licensed by a certified agency Phone-based: unable to observe nonverbal communication :( We are required to provide one if necessary Have phone service interpreters available/used Face-to-face interpreter, call coordinator *ONLY a certified interpreter can interpret medical info and give consent!!!* A face-to-face interpreter is the best so they can read nonverbal language CRASH = culture, respect, assess/affirm differences, show sensitivity, humanity

BOX 22.2 COMMON ASSESSMENT FINDINGS ASSOCIATED WITH POSTPARTUM DEPRESSION

Loss of pleasure or interest in life Low mood, especially in the morning; sadness, tearfulness Exhaustion that is not relieved by sleep Feelings of guilt Weight loss Low energy Irritability Poor personal hygiene Constipation Being preoccupied and unfocused Indecisiveness Diminished concentration Anxiety Despair Compulsive thoughts Loss of libido Loss of confidence Sleep difficulties (insomnia) Loss of appetite Bleak and pessimistic view of the future Not responding to infant's cries or cues for attention Social isolation, not answering the door or the phone Feelings of failure as a mother

Newborns can be classified by their birth weight regardless of their gestational age as follows:

Low birth weight: >2,500 g (>5.5 lb) Very low birth weight: >1,500 g (>3.5 lb) Extremely low birth weight: >1,000 g (>2.5 lb)

tocolytics

Medications used to suppress uterine activity Allow for the use of antenatal glucocorticoids to accelerate fetal lung maturity Also allows time for maternal transport to tertiary facility with an NICU

*informed consent*

Most care given in a health care setting is covered by the initial consent for treatment signed when the person becomes a client at that office or clinic, or by the consent to treatment signed upon admission to the hospital or other inpatient facility - certain procedures, however, require a specific process of informed consent. Procedures that require informed consent include major and minor surgery; invasive procedures such as amniocentesis, internal fetal monitoring, lumbar puncture, or bone marrow aspiration; treatments placing the person at higher risk, such as chemotherapy or radiation therapy; procedures or treatments involving research; application of restraints; and photography involving the person. Generally, only people over the age of majority (18 years of age) can legally provide consent for health care. Because children are minors, the process of consent involves obtaining written permission from a parent or legal guardian. In cases requiring a signature for consent, usually the parent gives consent for care of children less than 18 years of age except in certain situations (see discussion that follows). - *Take Note! Never assume that the adult accompanying the child is the parent or legal guardian. Always clarify the relationship of the accompanying adult.* The informed consent process, which must be done before the procedure or specific care, addresses the legal and ethical requirement of informing the person about the procedure. It originates from the right of the child and family to direct their care and the ethical responsibility of health care providers to involve the child and family in health care decisions. Nurses should involve children and adolescents in the decision-making process to the extent possible, though the parent is still ultimately responsible for giving consent. The physician or advanced practitioner providing or performing the treatment and/or procedure is responsible for informing the child and family about the procedure and obtaining consent by providing a detailed description of the procedure or treatment, the potential risks and benefits, and alternative methods available. The nurse's responsibility related to informed consent includes the following: Ensuring that the consent form is completed with signatures from the client (or parents or legal guardians if the client is a child) Serving as a witness to the signature process Determining whether the client or parents or legal guardians understand what they are signing by asking them pertinent questions - laws vary from state to state. Nurses must become familiar with state laws as well as the policies and procedures of the health care agency. Treating children without obtaining proper informed consent violates their rights, and the physician and/or facility may be held liable for any damages

*Know what you would teach new parents regarding weight loss in the newborn. What would be an appropriate percentage of weight loss?* Feed baby 8-12 times (q2-3 hrs) Bowel movements Appropriate amount of wt loss in newborn: 10% of birth weight

Most often, newborns are weighed using a digital scale that reads the weight in grams. Typically, the term newborn weighs 2,500 to 4,000 g (5 lb, 8 oz to 8 lb, 14 oz). Birth weights less than 10% or more than 90% on a growth chart are outside the normal range and need further investigation. Weights taken at later times are compared with previous weights and are documented with regard to gain or loss on a nursing flow sheet. Newborns can lose up to 10% of their initial birth weight by 3 to 4 days of age secondary to loss of meconium, extracellular fluid, and limited food intake. This weight loss is usually regained by the 10th day of life Using the information about gestational age and then considering birth weight, newborns can also be classified as follows: Small for gestational age (SGA)—weight less than the 10th percentile on standard growth charts (usually >5.5 lb) Appropriate for gestational age (AGA)—weight between 10th and 90th percentiles Large for gestational age (LGA)—weight more than the 90th percentile on standard growth charts (usually >9 lb)

*Understand the clinical manifestations and management for hyperemesis gravidarum*

Nausea and vomiting during pregnancy severe enough to result in dehydration, acidosis, and weight loss. May require hospitalization; if untreated, can be fatal. - Hyperemesis gravidarum is a severe form of nausea and vomiting of pregnancy associated with significant costs and psychosocial impacts. At least 70% to 85% of women experience nausea and vomiting during their pregnancy. The term morning sickness is often used to describe this condition when symptoms are relatively mild. Studies have shown that nausea and vomiting of pregnancy is associated with improved fetal outcomes, such as lower rates of miscarriage. Such symptoms usually disappear after the first trimester. This mild form mostly affects the quality of life of the woman and her family, whereas the severe form - hyperemesis gravidarum - results in dehydration, weight loss, electrolyte imbalance, and the need for hospitalization. Unlike morning sickness, hyperemesis gravidarum is a complication of pregnancy characterized by persistent, uncontrollable nausea and vomiting that begins in the first trimester and causes dehydration, ketosis, and weight loss of more than 5% of prepregnancy body weight. Risk factors for hyperemesis include previous pregnancy complicated by hyperemesis, molar pregnancies, history of helicobacter pylori infection, multiple gestation, prepregnancy history of genitourinary disorders, clinical hyperthyroid disorders, and prepregnancy psychiatric diagnosis. Hyperemesis (uncontrollable vomiting) is estimated to occur in approximately 2% of pregnant women. The prevalence increases in molar pregnancies and multiple gestations. The peak incidence is at 8 to 12 weeks of pregnancy, and symptoms usually resolve by week 20 - *Take Note! Every pregnant woman needs to be instructed to report any episodes of severe nausea and vomiting or episodes that extend beyond the first trimester.*

*herpes mgmt* [3]

No cure exists, but antiviral drug therapy helps to reduce or suppress symptoms, shedding, and recurrent episodes. Advances in Tx with *acyclovir 400 mg PO TID for 7-10 days, famciclovir 250 mg PO TID for 7-10 days, and valacyclovir 1 g PO BID for 7-10 days* have resulted in an improved quality of life for those infected with HSV. However, these drugs neither eradicate latent virus nor affect the risk, frequency, or severity of recurrences after the drug is discontinued Suppressive therapy is recommended for individuals with 6+ recurrences per year. The natural course of the disease is for recurrences to be less frequent over time. Mgmt of genital herpes includes antiviral therapy Safety of antiviral therapy has not been established during pregnancy Disclosure of this lifelong viral infection is often a challenge for individuals living with genital herpes. Therapeutic mgmt includes counseling regarding the natural history of the disease, the risk of sexual and perinatal transmission, and the use of methods to prevent further spread. a few guidelines to delivering information in a time-limited environment: (a) use all available client reading materials (b) have another knowledgeable staff member in the office who can spend extra time with women who need it (c) refer clients to good and accurate websites such as ashastd.org (d) know the phone numbers of herpes support groups in your area (e) educate the client to abstain from all sexual activity until HSV lesions resolve (f) use good handwashing technique to prevent spread (g) educate that there is NO cure, and that practicing safe sex (using condoms) with every sex act is essential to prevent transmission (h) encourage all clients to inform their current sex partners that they have genital herpes and to inform future partners before initiating a sexual relationship. Use a sympathetic, nonjudgmental approach The nurse can state in clear terms that having herpes does not change the core of the person or make them less worthwhile

*Understand the causes of hypoxemia and how it relates to decelerations in the FHR* Fetal compromise

Nonreassuring FHR patterns associated with hypoxemia Hypoxemia can deteriorate to severe fetal hypoxia (Inadequate supply of oxygen at cellular level) Non reassuring FHR includes (don't want to see): Baseline FHR <110 beats/min or >160 bpm Absent or persistently minimal variability Recurrent late or variable decelerations Bradycardia Causes of Decreased variability: Fetal hypoxemia & acidosis

*nursing care for preterm labor!*

Note frequency and intensity of contractions Encourage adherence to bedrest Monitor I & O Fluid restriction of 1500-2500mL (esp. for Brethine & Mag) Assess heart and lung sounds and DTR's Notify personnel if birth suspected Increased risk for pulmonary edema Monitor fetal heart rate for fetal well-being and signs of distress. Provide emotional support

*BOX 14.1 INTERVENTIONS FOR CATEGORY III PATTERNS*

Notify the health care provider about the pattern and obtain further orders, making sure to document all interventions and their effects on the FHR pattern. Discontinue oxytocin or other uterotonic agent as dictated by the facility's protocol, if it is being administered. Turn the client on her left or right lateral, knee-chest, or hands and knees to increase placental perfusion or relieve cord compression. Administer oxygen via nonrebreather face mask to increase fetal oxygenation. Increase the intravenous fluid rate to improve intravascular volume and correct maternal hypotension. Assess the client for any underlying contributing causes. Provide reassurance that interventions are to effect pattern change. Modify pushing in the second stage of labor to improve fetal oxygenation. Document any and all interventions and any changes in FHR patterns. Prepare for an expeditious surgical birth if the pattern is not corrected in 30 minutes.

*ways the nurse can provide family centered care* [5]

Nurse may have role as a gatekeeper - advocates for the pt, like when the mom is tired in L & D and a lot of family is there Ex: a lot of family members in prenatal visits, some people have decision making roles (aka not the mother) so must include them in education, etc May take care of pt adopting infant, do things that facilitate bonding and attachment Childbearing is a family event: 1. Include family members in ALL aspects of care 2. Childbirth prep. for parents, siblings, & others 3. Involvement of SO in birthing process 4. Sibling visitation 5. Strategies to foster family members' attachment to newborn

*Nursing Management R/T Cesarean sections*

Once the decision has been made to proceed with a cesarean birth, assess the woman's knowledge of the procedure and necessary preparation. Assist with obtaining diagnostic tests as ordered. These tests are usually ordered to ensure the well-being of both parties and may include a complete blood count; urinalysis to rule out infection; blood type and cross-match so that blood is available for transfusion if needed; an ultrasound to determine fetal position and placental location; and an amniocentesis to determine fetal lung maturity if needed. Although the nurse's role in a cesarean birth can be very technical and skill oriented at times, the focus must remain on the woman, not the equipment surrounding the bed. Care should be centered on the family, not the surgery. Provide education and minimize separation of the mother, father, and newborn. Remember that the client is anxious and concerned about her welfare as well as that of her child. Use touch, eye contact, therapeutic communication, and genuine caring to provide couples with a positive birth experience, regardless of the type of delivery.

COMPARISON CHART 19.1 PLACENTA PREVIA VERSUS ABRUPTIO PLACENTAE Manifestation Placenta Previa (P) Abruptio Placentae (A)

Onset P: Insidious A: Sudden Type of bleeding P: Always visible; slight, then more profuse A: Can be concealed or visible Blood description P: Bright red A: Dark Discomfort/pain P: None (painless) A: Constant; uterine tenderness on palpation Uterine tone P: Soft and relaxed A: Firm to rigid FHR P: Usually in normal range A: Fetal distress or absent Fetal presentation P: May be breech or transverse lie; engagement is absent A: No relationship

*Review the insulin needs for a diabetic pregnant woman* Tidwell, Antepartum Complications,

Over the course of pregnancy, insulin resistance does change. It peaks in the last trimester to provide more nutrients to the fetus. The insulin resistance typically results in postprandial hyperglycemia, although some women also have an elevated fasting blood glucose level. With this increased demand on the pancreas in late pregnancy, women with diabetes or glucose intolerance cannot accommodate the increased insulin demand; glucose levels rise as a result of insulin deficiency, leading to hyperglycemia. Subsequently, the mother and her fetus can experience major problems. • Insulin therapy or oral hypoglycemic agents, in addition to diet and exercise, are main elements of achieving glycemic control • In 1st trimester, insulin doses may be reduced to prevent hypoglycemia resulting from increased insulin sensitivity • Diabetes Mellitus effect on the fetus: macrosomia • As the pregnancy progresses, there may changes in insulin resistance • *Fasting blood glucose level: less than 92mg/dL* • At 1 hour after eating: < 180 • At 2 hours after eating: < 153 • At 3 hours after eating: < 140 • Rescreen at 24-28wks gestation to check for DM development • A1C should be <7%

Teaching Guidelines 24.1 CARING FOR YOUR NEWBORN AT HOME R/T NAS

Position your newborn with the head elevated to prevent choking. To aid your newborn's sucking and swallowing during feeding, position the chin downward and support it with your hand. Place your newborn on his or her back to sleep or nap, never on the stomach. Keep a bulb syringe close by to suction your newborn's mouth in case of choking. Cluster newborn care (bathing, feeding, dressing) to prevent overstimulation. If your newborn is fussy or crying, try these measures to help calm him or her: Wrap your newborn snugly in a blanket and gently rock in rocking chair. Take the baby for a ride in the car (using a newborn car seat). Play soothing music and "dance" with the newborn. Use a wind-up swing with music. To help your newborn get to sleep, try these measures: Schedule a bath with a gentle massage prior to bedtime. Change diaper and clothes to make the baby comfortable. Feed the baby just prior to bedtime. If the newborn cries when put in crib and all needs are met, allow him or her to cry. Use a rocking chair to feed and sing a soft lullaby. Call your primary care provider if you observe withdrawal behaviors such as: Slight tremors (shaking) of hands and legs Stiff posture when held in your arms Irritability and frequent fussiness High-pitched cry, excessive sucking motions Erratic sleep pattern Frequent yawning, nasal stuffiness, sweating Prolonged time needed to feed Frequent vomiting after feeding

*Non-stress test (NST)* Want to see an acceleration with fetal movement Want to see at least 3 accelerations Negative result: if we don't see them (that's bad) If non-reactive, HCP will order BPP, Comprehensive exam that gives us specific information! Ultrasound is not good enough nursing implications and pt teaching

Used *MOST* to determine fetal well being Normal FHR should increase with fetal movement if CNS intact May be blunted by: 1. Hypoxia 2. Acidosis 3. Drugs (analgesics, barbiturates, & beta blockers) 4. Fetal sleep 5. Some congenital anomalies *nursing mgmt* • Empty bladder • Place pt in reclining chair or semi Fowler position • *Attach external fetal monitoring doppler transducer (fetal HR) and tocodynamometer (uterine contractions)* • *Obtain baseline strip (15-30min)* • Patient uses marker to record fetal movement • Observe tracing for fetal accelerations in response to movements over 20-30 minutes • goal: reactive result • *2 or more accelerations of 15 beats/min lasting for 15 seconds over a 20 minute period* • Normal baseline rate & Moderate variability • If test doesn't meet criteria over 40 minutes considered nonreactive, Further testing is indicated: CST or BPP

PROVIDING POSTOPERATIVE CARE R/T Cesarean Sections

Postoperative care for the mother who has had a cesarean delivery is similar to that for one who has had a vaginal birth, with a few additional measures. Assess vital signs and lochia flow every 15 minutes for the first hour, then every 30 minutes for the next hour, and then every 4 hours if stable. Assist with perineal care and instruct the client in the same. Inspect the abdominal dressing and document description, including any evidence of drainage. Assess uterine tone to determine fundal firmness. Check the patency of the intravenous line, making sure the infusion is flowing at the correct rate. Inspect the infusion site frequently for redness. Assess the woman's level of consciousness if sedative drugs were administered. Institute safety precautions until the woman is fully alert and responsive. If a regional anesthetic was used, monitor for the return of sensation to the legs. Assess for evidence of abdominal distention and auscultate bowel sounds. Assist with early ambulation to prevent respiratory and cardiovascular problems and to promote peristalsis. Monitor intake and output at least every 4 hours initially and then every 8 hours as indicated. Encourage the woman to cough, perform deep-breathing exercises, and use the incentive spirometer every 2 hours. Enhance comfort and general well-being. Administer analgesics as ordered and provide comfort measures, such as splinting the incision and pillows for positioning. Assist the client to move in bed and turn side to side to improve circulation. Also encourage the woman to ambulate to promote venous return from the extremities. Prevent/minimize postoperative complications. - Encourage early touching and holding of the newborn to promote bonding. Promote family unity and bonding. Assist with breast-feeding initiation and offer continued support. Suggest alternate positioning techniques to reduce incisional discomfort while breast-feeding. (See Chapter 18 for breast-feeding positions.) Review with the couple their perception of the surgical birth experience. Allow them to verbalize their feelings and assist them in positive coping measures. Promote a positive emotional response to the birth experience and parenting role. Prior to discharge, teach the woman about the need for adequate rest, activity restrictions such as lifting, and signs and symptoms of infection. Provide information about postpartum care at home upon discharge.

*Identify risk factors associated with postpartum hemorrhage.* BOX 16.1: FACTORS INCREASING A WOMAN'S RISK FOR POSTPARTUM COMPLICATIONS Risk Factors for Postpartum Hemorrhage

Precipitous labor (less than 3 hours) Uterine atony Placenta previa or abruptio placenta Labor induction or augmentation Operative procedures (vacuum extraction, forceps, cesarean birth) Retained placental fragments Prolonged third stage of labor (more than 30 minutes) Multiparity, more than three births closely spaced Uterine overdistention (large infant, twins, hydramnios)

*placental abruption clinical manifestations* *abruptio placentae*

Premature detachment of a normally situated placenta. premature separation of a normally implanted placenta from the maternal myometrium. occurs in about 1% of all pregnancies throughout the world and is associated with significant perinatal mortality and morbidity. Risk factors: preeclampsia, gestational hypertension, seizure activity, advanced maternal age >34, uterine rupture, trauma, smoking, cocaine use, coagulation defects, chorioamnionitis, premature rupture of membranes, hydramnios, uterine trauma, external cephalic version for breech presentation, previous history of abruption, domestic violence, and placental pathology. These conditions may force blood into the underlayer of the placenta and cause it to detach Take Note!: *Classic manifestations of abruptio placentae include painful, dark-red vaginal bleeding (port-wine color) because the bleeding comes from the clot that was formed behind the placenta; "knife-like" abdominal pain; uterine tenderness; contractions; and decreased fetal movement. Rapid assessment is essential to ensure prompt, effective interventions to prevent maternal and fetal morbidity and mortality.* Take Note!: *Vital signs can be within normal range, even with significant blood loss, because a pregnant woman can lose up to 40% of her total blood volume without showing signs of shock*

*Signs of resp. distress (from notes)*

RR >60 or <30 apnea >20 seconds nasal flaring retractions grunting See-saw breathing which is described as: Chest and abdomen rises/falls simultaneously, Working harder to get air in and out using accessory muscles

Teaching Guidelines 19.2 TEACHING FOR THE WOMAN WITH MILD PREECLAMPSIA

Rest in a quiet environment to prevent cerebral disturbances. Drink 8 to 10 glasses of water daily. Consume a balanced, high-protein diet including high-fiber foods. Obtain intermittent bed rest to improve circulation to the heart and uterus. Limit your physical activity to promote urination and subsequent decrease in blood pressure. Enlist the aid of your family so that you can obtain appropriate rest time. Perform self-monitoring as instructed, including: Taking your own blood pressure twice daily Checking and recording weight daily Performing urine dipstick twice daily Recording the number of fetal kicks daily Contact the home health nurse if any of the following occurs: Increase in blood pressure Protein present in urine Gain of more than 1 pound in 1 week Burning or frequency when urinating Decrease in fetal activity or movement Headache (forehead or posterior neck region) Dizziness or visual disturbances Increase in swelling in hands, feet, legs, and face Stomach pain, excessive heartburn, or epigastric pain Decreased or infrequent urination Contractions or low back pain Easy or excessive bruising Sudden onset of abdominal pain Nausea and vomiting

Nursing Assessment R/T Cesarean Sections

Review the woman's history for indications associated with cesarean birth and complete a physical examination. Any condition that prevents the safe passage of the fetus through the birth canal or that seriously compromises maternal or fetal well-being may be an indication for a cesarean birth. Controversy exists over the option of elective cesarean birth on maternal request. The Agency for Healthcare Research and Quality (AHRQ) has published a report on maternal request for a surgical birth, and although there is not high-quality evidence to support this, it is recognized that women have the right to be actively involved in choosing the route of her childbirth. The pregnant woman requesting a surgical birth must be made aware of the associated risks and benefits for the current and any subsequent pregnancies is reasonable. The clinician's role should be to provide the best evidence-based counseling possible to the woman and to respect her autonomy and decision-making capabilities when considering route of birth. Examples of specific indications include *active genital herpes, fetal macrosomia, fetopelvic disproportion, prolapsed umbilical cord, placental abnormality (placenta previa or abruptio placentae), previous classic uterine incision or scar, gestational hypertension, diabetes, positive human immunodeficiency virus (HIV) status, and dystocia.* Fetal indications include *malpresentation (nonvertex presentation), congenital anomalies (fetal neural tube defects, hydrocephalus, abdominal wall defects), and fetal distress*

*Rh Incompatibility (from the book)*

Rh incompatibility is a condition that develops when a woman with Rh-negative blood type is exposed to Rh-positive blood cells and subsequently develops circulating titers of Rh antibodies. Individuals with Rh-positive blood type have the D antigen present on their red cells, whereas individuals with an Rh-negative blood type do not. The presence or absence of the Rh antigen on the RBC membrane is genetically controlled. In the United States, about 15% of the White population, 5% to 8% of the African American and Hispanic populations, and 1% to 2% of the Asian and Native American populations are Rh negative. The vast majority (85%) of individuals is considered Rh positive. Rh incompatibility most commonly arises with exposure of an Rh-negative mother to Rh-positive fetal blood during pregnancy or birth, during which time erythrocytes from the fetal circulation leak into the maternal circulation. Isoimmunization can also occur during an amniocentesis, ectopic pregnancy, placenta previa, placenta abruption, in utero fetal death, spontaneous abortion, or abdominal/pelvic trauma. After a significant exposure, alloimmunization or sensitization occurs. As a result, maternal antibodies are produced against the foreign Rh antigen. Theoretically, fetal blood and maternal blood do not mix during pregnancy. In reality, however, small placental accidents (transplacental bleeds secondary to minor separation), abortions, ectopic pregnancy, abdominal trauma, trophoblastic disease, amniocentesis, placenta previa, and abruptio placentae allow fetal blood to enter the maternal circulation and initiate the production of antibodies to destroy Rh-positive blood. The amount of fetal blood necessary to produce Rh incompatibility varies. In one study, less than 1 mL of Rh-positive blood was shown to result in sensitization of women who are Rh negative. Once sensitized, it takes approximately a month for Rh antibodies in the maternal circulation to cross over into the fetal circulation. In 90% of cases, sensitization occurs during delivery. Thus, most firstborn infants with Rh-positive blood type are not affected because the short period from first exposure of Rh-positive fetal erythrocytes to the birth of the infant is insufficient to produce a significant maternal IgG antibody response. The risk and severity of alloimmune response increase with each subsequent pregnancy involving a fetus with Rh-positive blood. A second pregnancy with an Rh-positive fetus often produces a mildly anemic infant, whereas succeeding pregnancies produce infants with more serious hemolytic anemia.

*mgmt for severe preeclampsia*

Severe preeclampsia may develop suddenly and bring with it high blood pressure of more than 160/110 mm Hg, proteinuria of more than 5 g in 24 hours, oliguria of less than 400 mL in 24 hours, cerebral and visual symptoms, and rapid weight gain. This clinical picture signals severe preeclampsia, and immediate hospitalization is needed. Treatment is highly individualized and based on disease severity and fetal age. Birth of the infant is the only cure, because preeclampsia depends on the presence of trophoblastic tissue. Therefore, the exact age of the fetus is assessed to determine viability. Severe preeclampsia is treated aggressively because hypertension poses a serious threat to mother and fetus. The goal of care is to stabilize the mother-fetus dyad and prepare for birth. Therapy focuses on controlling hypertension, preventing seizures, preventing long-term morbidity, and preventing maternal, fetal, or newborn death. Intense maternal and fetal surveillance starts when the mother enters the hospital and continues throughout her stay. The woman in labor with severe preeclampsia typically receives oxytocin to stimulate uterine contractions and magnesium sulfate to prevent seizure activity. Oxytocin and magnesium sulfate can be given simultaneously via infusion pumps to ensure both are administered at the prescribed rate. Magnesium sulfate is given intravenously via an infusion pump. A loading dose of 4 to 6 g is given over 5 minutes. Then, a maintenance dose of 2 g/hr is given. The client is evaluated closely for magnesium toxicity. If at all possible, a vaginal delivery is preferable to a cesarean birth for better maternal outcomes and less risk associated with a surgical birth. PGE2 gel may be used to ripen the cervix. A cesarean birth may be performed if the client is seriously ill. A pediatrician/neonatologist or neonatal nurse practitioner should be available in the birthing room to care for the newborn. A newborn whose mother received high doses of magnesium sulfate needs to be monitored for respiratory depression, hypocalcemia, and hypotonia. Decreased fetal heart rate variability may occur but, in general, magnesium sulfate does not pose a risk to the fetus. The newborn may exhibit respiratory depression, loss of reflexes, muscle weakness, and neurologic depression

Engorgement may occur as the milk comes in around day 3 or 4 after birth of the newborn. Explain to the mother that engorgement, though uncomfortable, is self-limited and will resolve as the newborn continues to nurse. The mother should continue to nurse during engorgement to avoid a plugged milk duct, which could lead to mastitis. Provide the following tips for relieving engorgement: idk if engorgement will be on final or not

Take warm to hot showers to encourage milk release. Express some milk manually before breast-feeding. Wear a supportive nursing bra 24 hours a day to provide support. Feed the newborn in a variety of positions—sitting up and then lying down. Massage the breasts from under the axillary area down toward the nipple. Increase the frequency of feedings. Apply warm compresses to the breasts prior to nursing. Stay relaxed while breast-feeding. Use a breast pump if nursing or manual expression is not effective. Remember that this condition is temporary and resolves quickly.

*Explain what would be appropriate for the nurse to instruct a patient with Ruptured Membranes.*

Tell her to come into the hospital so you can evaluate her with your eyes and hands. • A ruptured ectopic pregnancy is a *medical emergency*; therefore, prediction of any tubal rupture before its occurrence is extremely important • Rupture of membranes with loss of amniotic fluid prior to the onset of labor is termed prelabor rupture of membranes (PROM). • It occurs in 8% to 10% of women with term pregnancies, the majority of whom will begin labor spontaneously within 24 hours. • The rupture of membranes can result in either a sudden gush or a steady leakage of amniotic fluid. • Although much of the amniotic fluid is lost when the rupture occurs, a continuous supply is produced to ensure protection of the fetus until birth. • After the amniotic sac has ruptured, the barrier to infection is gone and an ascending infection is possible. • In addition, there is a danger of cord prolapse if engagement has not occurred with the sudden release of fluid and pressure with rupture. • *Due to the possibility of these complications, advise women to notify their HCP and go in for an evaluation.*

*Describe the structure and function of the umbilical cord.* Composed of 2 arteries and 1 vein Has Wharton's jelly Function: *carries blood and nutrients from the placenta and back*

The *definitive connecting stalk b/w the embryo or fetus and the placenta* • at birth it is primarily composed of *mucoid connective tissue (Wharton jelly)* in which the umbilical vessels are embedded • The umbilical vein carries oxygen-rich blood from the placenta to the liver and through the ductus venosus • cord is formed from the amnion while the placenta is developing (end of the 2nd week) • It's the lifeline from the mother to the growing embryo • *It contains 1 large Vein and 2 small Arteries.* • *Wharton's jelly* (a specialized connective tissue) surrounds these 3 blood vessels in the umbilical cord to prevent compression, *which would cut off fetal blood and nutrient supply* (effect of supine hypotension on fetus) • The cord reaches its maximum length by 30 weeks of gestation • Its length is determined by genetics and intrauterine space and fetal activity, which places tension on the cord • At term, the average umbilical cord is 22 inches long and about 1 inch wide

*Herpes* Viral not bacterial No cure Tx: acyclovir is the most common Outbreak and wants to have sex? Ideally, no sex But if they must, use a condom to minimize transmission *Herpes s/s* [11] [5] Genital Herpes Simplex (HSV) Clinical manifestations can be divided into the primary (1st) episode and recurrent infections.

The 1st or primary episode is usually the most severe, with a prolonged period of viral shedding. Primary HSV is a systemic disease characterized by: 1. multiple painful vesicular lesions 2. mucopurulent discharge 3. superinfection with candida 4. fever 5. chills 6. malaise 7. dysuria 8. HA 9. genital irritation 10. inguinal tenderness 11. lymphadenopathy. The lesions in the primary herpes episode are frequently located on the vulva, vagina, and perineal areas. The vesicles will open and weep and finally crust over, dry, and disappear without scar formation. This viral shedding process usually takes up to 2 weeks to complete. Recurrent infection episodes are usually much milder and shorter in duration than the primary. 1. Tingling 2. itching 3. pain 4. unilateral genital lesions 5. a more rapid resolution of lesions Recurrent herpes is a localized disease characterized by typical HSV lesions at the site of initial viral entry. Recurrent herpes lesions are fewer in number and less painful and resolve more rapidly. Diagnosis of HSV is often based on clinical s/s and is confirmed by viral culture of fluid from the vesicle. Pap smears are an insensitive and nonspecific diagnostic test for HSV and should not be relied on for diagnosis. The woman should be tested for all common STIs, especially if she has a new sexual partner. Hopefully, the woman would initiate an open conversation with her sexual partner about the risk of transmission and the need for safer sexual practices M: Spontaneous abortion, intrauterine infection, preterm labor, PROM, fetal growth restriction F: Birth anomalies; transplacental infection

*placenta accrete*

The abnormal adherence of the chorionic villi to the myometrium, associated with partial or complete absence of the decidua basalis and, in particular, the stratum spongiosum. is a potentially life-threatening obstetrical hemorrhagic condition that requires a multidisciplinary approach to management. The incidence has increased and seems to parallel the increasing cesarean birth rate or intrauterine procedures. is a condition in which the placenta attaches itself too deeply into the wall of the uterus but does not penetrate the uterine muscle. It is further subcategorized as placenta increta, when the placenta invades the myometrium, and placenta percreta, when it has extended through the myometrium and uterine serosa and adjacent tissue. A common risk of placenta accreta during the birthing process is the possibility of hemorrhaging during manual attempts to detach the placenta. 1 in 530 births results in this condition. The specific cause of placenta accreta is unknown, but it can be related to placenta previa, advanced maternal age, smoking, and previous cesarean births. According to the literature, a cesarean birth increases the possibility of a future placenta accreta; the more cesarean births that are done, the greater the incidence. Postpartum hemorrhage is a complication associated with placenta accreta. Ninety percent of accretas have postpartum hemorrhage, and 50% of these will result in a hysterectomy. Women at highest risk of emergency hysterectomy are those who are multiparous, had a cesarean birth in either a previous or the present pregnancy, or had abnormal placentation. The essential management issues are early detection and immediate and appropriate intervention. If a placenta accreta diagnosis is made, the client should be counseled that a cesarean section and possible hysterectomy may be necessary interventions. Placenta accreta is typically diagnosed after birth when the placenta fails to normally separate from the uterine wall. A prenatal screening diagnosis via ultrasound and MRI would decrease maternal and fetal morbidities and mortalities. A profuse hemorrhage may result because the uterus cannot contract to close off the open blood vessels. Management will depend on the severity of the bleeding and frequently necessitates a prompt hysterectomy. Nurses need to be prepared to assist in this emergency situation as dictated by the health care provider.

*ANTHROPOID PELVIS*

The anthropoid pelvis is common in men and is most common in non-White women • It occurs in approximately 25% of women • The pelvic inlet is oval and the sacrum is long, producing a deep pelvis (wider front to back [anterior to posterior] than side to side [transverse]). • vaginal birth is more favorable with this pelvic shape compared with the android or platypelloid shape

Preventative Care

The concept of prevention is a key part of maternal and pediatric nursing. The emphasis on health care delivery has moved beyond primary preventive health care (e.g., well-child checkups, routine physical examinations, prenatal care, and treatment of common acute illnesses) and now encompasses secondary and tertiary care. All health professionals have a special role in health promotion, health protection, and disease prevention. Much of nursing involves prevention, early identification, and prompt treatment of health problems and monitoring for emerging threats that might lead to health problems. Nurses provide health care for women and children at all three levels of prevention. This care often involves advocacy for services to meet their needs. FIGURE 2.3. Levels of prevention. A. At the primary prevention level, *the nurse provides anticipatory guidance and family teaching.* B. At the secondary level of prevention, *a woman undergoes a mammography screening for early detection of breast problems.* C. At the tertiary level of prevention, *a child with a developmental disability participates in a rehabilitation program.*

*placenta previa*

The condition in which the placenta is implanted in the lower segment of the uterus, extending to the margin of the internal os of the cervix or partially or completely obstructing the os. SYN placental presentation is a bleeding condition that occurs during the last two trimesters of pregnancy. In placenta previa (literally, "afterbirth first"), the placenta implants over the cervical os. It may cause serious morbidity and mortality to the fetus and mother. The risk of placenta previa in a first pregnancy is 1 in 400, but it rises to 1 in 160 after one cesarean section; 1 in 60 after two; 1 in 30 after three; and 1 in 10 after four cesarean sections and is associated with potentially serious consequences from hemorrhage, abruption (separation) of the placenta, or emergency cesarean birth. With the rising incidence of cesarean section operations combined with increasing maternal age and more infertility treatments, the number of cases of placenta previa is increasing dramatically. The cesarean section rate must be reduced to decrease maternal morbidity and mortality. Comprehensive risk assessment, combined with advances in ultrasound, can provide earlier detection of this impaired placental implantation Total placenta previa: the internal cervical os is completely covered by the placenta Partial placenta previa: the internal os is partially covered by the placenta Marginal placenta previa: the placenta is at the margin or edge of the internal os Low-lying placenta previa: the placenta is implanted in the lower uterine segment and is near the internal os but does not reach it

*Assessing Uterine Contractions*

The primary power of labor is uterine contractions, which are involuntary. Uterine contractions increase intrauterine pressure, causing tension on the cervix. This tension leads to cervical dilation and thinning, which in turn eventually forces the fetus through the birth canal. Normal uterine contractions have a contraction (systole) and a relaxation (diastole) phase. The contraction resembles a wave, moving downward to the cervix and upward to the fundus of the uterus. Each contraction starts with a building up (increment), gradually reaching an acme (peak intensity), and then a letting down (decrement). Each contraction is followed by an interval of rest, which ends when the next contraction begins. At the acme (peak) of the contraction, the entire uterus is contracting, with the greatest intensity in the fundal area. The relaxation phase follows and occurs simultaneously throughout the uterus. Uterine contractions during labor are monitored by palpation and by electronic monitoring. Assessment of the contractions includes frequency, duration, intensity, and uterine resting tone. Uterine contractions with intensity of 30 mm Hg or greater initiate cervical dilation. During active labor, the intensity usually reaches 50 to 80 mm Hg. Resting tone is normally between 5 and 10 mm Hg in early labor and between 12 and 18 mm Hg in active labor. To palpate the fundus for contraction intensity, place the pads of your fingers on the fundus and describe how it feels: like the tip of the nose (mild), like the chin (moderate), or like the forehead (strong). Palpation of intensity is a subjective judgment of the indentability of the uterine wall; a descriptive term is assigned (mild, moderate, or strong) The second method used to assess the intensity of uterine contractions is electronic monitoring, either external or internal. Both methods provide a reasonable measurement of the intensity of uterine contractions. Although the external fetal monitor is sometimes used to estimate the intensity of uterine contractions, it is not as accurate an assessment tool. Take Note!: *Frequent clinical experience is needed to gain accuracy in assessing the intensity of uterine contractions*

*patient teaching for Semen Analysis* Assessing Male Factors R/T infertility

The initial screening evaluation for the male partner should include a reproductive history and a *semen analysis*. From the male perspective, 3 things must happen for conception to take place: 1. the number of sperm must be adequate 2. those sperm must be healthy and mature 3. the sperm must be able to penetrate and fertilize the egg Normal males have more than *20 million sperm per milliliter with >50% motility!* Semen analysis is the *most important indicator of male fertility!* *The man should abstain from sexual activity for 24 to 48 hours before giving the sample!* Man is asked to produce a specimen by ejaculating into a specimen container and delivering it to the laboratory for analysis within 1-2 hours. Specimen s analyzed for volume, viscosity, number of sperm, sperm viability, motility, and sperm shape. If semen parameters are normal, no further male evaluation is necessary *If sperm "clump" together suspect & check for infection!* A recent study shows that social strain and stress are highest among couples without a clear etiology for their infertility. These findings highlight the clinically significant negative sexual, personal, and social strains of a perceived infertility diagnosis for men. Nurses need to be very cognizant of this impact on males and address it. The physical examination routinely includes: 1. Assessment for appropriate male sexual characteristics, such as body hair distribution, development of the Adam's apple, and muscle development 2. Examination of the penis, scrotum, testicles, epididymis, and vas deferens for abnormalities (e.g., nodules, irregularities, varicocele) 3. Assessment for normal development of external genitalia (small testicles) 4. Performance of a digital internal examination of the prostate to check for tenderness or swelling

*How to identify ROM*

The integrity of the membranes can be determined during the vaginal examination. Typically, if intact, the membranes will be felt as a soft bulge that is more prominent during a contraction. If the membranes have ruptured, the woman may have reported a sudden gush of fluid. Membrane rupture also may occur as a slow trickle of fluid. When membranes rupture, the priority focus should be on assessing fetal heart rate (FHR) first to identify a deceleration, which might indicate cord compression secondary to cord prolapse. If the membranes are ruptured when the woman comes to the hospital, the health care provider should ascertain when it occurred. Prolonged ruptured membranes increase the risk of infection as a result of ascending vaginal pathologic organisms for both mother and fetus. Signs of intrauterine infection to be alert for include maternal fever, fetal and maternal tachycardia, foul odor of vaginal discharge, and an increase in white blood cell count. To confirm that membranes have ruptured, a sample of fluid is taken from the vagina via a nitrazine yellow dye swab to determine the fluid's pH. Vaginal fluid is acidic, whereas amniotic fluid is alkaline and turns a nitrazine swab blue. Sometimes, however, false-positive results can occur, especially in women experiencing a large amount of bloody show, because blood is alkaline. The membranes are most likely intact if the nitrazine swab remains yellow to olive green, with pH between 5 and 6. The membranes are probably ruptured if the nitrazine swab turns a blue-green to deep blue, with pH ranging from 6.5 to 7.5 (Tharpe, Farley, & Jordan, 2016). If the nitrazine test is inconclusive, an additional test, called the fern test, can be used to confirm rupture of membranes. With this test, a sample of vaginal fluid is obtained, applied to a microscope slide, and allowed to dry. Using a microscope, the slide is examined for a characteristic fern pattern that indicates the presence of amniotic fluid.

*What are the nursing interventions of a newborn with NAS Neonatal Abstinence Syndrome?* Nursing Management

The needs of the substance-exposed newborn are multiple, complex, and costly, both to the health care system and to society. Substance abuse takes place among people of all colors, sizes, shapes, incomes, types, and conditions. Most pregnant women are unaware of the adverse impact their substance abuse can have on the newborn. Pregnant women dependent on opioids are maintained on methadone as the current standard of care, which provides multiple benefits including improved prenatal care, reduced fetal mortality, and improved fetal growth. - Nurses are in a unique position to help because they interact with high-risk mothers and newborns in many settings, including the community, health care facilities, and family agencies. It is the responsibility of all nurses to identify, educate, counsel, and refer pregnant women with substance-abusing problems. Ex: nurses can be instrumental in increasing the number of pregnant women who make a serious attempt to quit smoking by using the "5 A's" approach: Ask: Ask all women if they smoke and would like to quit. Advise: Encourage the use of clinically proven treatment plans. Assess: Provide motivation by discussing the "5 Rs": Relevance of quitting to the woman Risk of continued smoking to the fetus Rewards of quitting for both Roadblocks to quitting Repeat at every visit Assist: Help the woman to protect her fetus and newborn from the negative effects of smoking. Arrange: Schedule follow-up visits to reinforce the woman's commitment to quit. Although this approach is geared to smoking cessation, nurses can adapt it to focus on cessation for any substance use. Early, supportive, ongoing nursing care is critical to the well-being of the mother and her newborn. Nurses have an ethical responsibility to provide evidence-based and nonjudgmental care to this highly vulnerable population. Caring for a substance-exposed newborn remains a major challenge to HCPs. *The major goals include providing comfort to the newborn by relieving symptoms, improving feeding and weight gain, preventing seizures, promoting mother-newborn interactions, and reducing the incidence of newborn mortality and abnormal development*

*intervening for eclampsia*

The onset of seizure activity identifies eclampsia. Typically, eclamptic seizures are generalized and start with facial twitching. The body then becomes rigid, in a state of tonic muscular contraction. The clonic phase of the seizure involves alternating contraction and relaxation of all body muscles. Respirations stop during seizure activity and resume shortly after it ends. Client safety is the primary concern during eclamptic seizures. If possible, turn the client to her side and remain with her. Make sure that the side rails are up and padded. Dim the lights and keep the room quiet. Document the time and sequence of events as soon as possible. After the seizure activity has ceased, suction the nasopharynx as necessary and administer oxygen. Continue the magnesium sulfate infusion to prevent further seizures. Ensure continuous electronic fetal monitoring, evaluating fetal status for changes. Also assess the client for uterine contractions. After the client is stabilized, prepare her for the birthing process as soon as possible to reduce the risk of perinatal mortality.

*What type of medications would be given in a patient with a laceration?* o Med to numb the area, ointment o Stool softener, Colace

The perineum is often edematous and bruised for the first day or two after birth. If the birth involved an episiotomy or laceration, complete healing may take as long as 4 to 6 months in the absence of complications at the site, such as hematoma or infection. The muscle tone may or may not return to normal, depending on the extent of injury to muscle, nerve, and connecting tissues. Perineal lacerations may extend into the anus and cause considerable discomfort for the mother when she is attempting to defecate or ambulate. The presence of swollen hemorrhoids may also heighten discomfort. Local comfort measures such as ice packs, pouring warm water over the area via a peribottle, witch hazel pads, anesthetic sprays, and sitz baths can relieve pain.

*mgmt of HELLP*

The treatment for HELLP syndrome is based on the severity of the disease, the gestational age of the fetus, and the condition of the mother and fetus. The mainstay of treatment is lowering of high blood pressure with rapid-acting antihypertensive agents, prevention of convulsions or further seizures with magnesium sulfate, and use of steroids for fetal lung maturity if necessary, followed by the birth of the infant and placenta (Foley et al., 2014). The client should be admitted or transferred to a tertiary center with a neonatal intensive care unit. Additional treatment includes correction of the coagulopathies that accompany HELLP syndrome. After this syndrome is diagnosed and the woman's condition is stable, birth of the infant is indicated. Magnesium sulfate is used prophylactically to prevent seizures. Antihypertensives such as hydralazine or labetalol are given to control blood pressure. Blood component therapy - such as fresh-frozen plasma, packed red blood cells, or platelets - is transfused to address the microangiopathic hemolytic anemia. Birth may be delayed up to 96 hours so that betamethasone or dexamethasone can be given to stimulate lung maturation in the preterm fetus. - Nursing management of the woman diagnosed with HELLP syndrome is the same as that for the woman with severe preeclampsia. If possible, the woman with HELLP syndrome should be transferred to a tertiary care center once she has been assessed and stabilized. Closely monitor the client for changes and provide ongoing support throughout this experience.

*intervening for preeclampsia*

The woman with mild preeclampsia requires frequent monitoring to detect changes because preeclampsia can progress rapidly. Instruct all women in the signs and symptoms of preeclampsia and urge them to contact their health care professional for immediate evaluation should any occur. Typically, women with mild preeclampsia can be managed at home if they have a good understanding of the disease process, blood pressure and vital signs are stable, there are no abnormal laboratory test results, and if good fetal movement is demonstrated. The home care nurse makes frequent visits and follow-up phone calls to assess the woman's condition, to assist with scheduling periodic evaluations of the fetus (such as nonstress tests), and to evaluate any changes that might suggest a worsening of the woman's condition. - Early detection and management of mild preeclampsia is associated with the greatest success in reducing progression of this condition. As long as the client carries out the guidelines of care as outlined by the health care provider and remains stable, home care can continue to maintain the pregnancy until the fetus is mature. If disease progression occurs, hospitalization is required.

*intervening for severe preeclampsia*

The woman with severe preeclampsia requires hospitalization. Maintain the client on complete bed rest in the left lateral lying position. Ensure that the room is dark and quiet to reduce stimulation. Give sedatives as ordered to encourage quiet bed rest. The client is at risk for seizures if the condition progresses. Therefore, institute and maintain seizure precautions, such as padding the side rails and having oxygen, suction equipment, and call light readily available to protect the client from injury. - Closely monitor the client's blood pressure. Administer antihypertensives as ordered to reduce blood pressure (Drug Guide 19.3). Assess the client's vision and level of consciousness. Report any changes and any complaints of headache or visual disturbances. Offer a high-protein diet with 8 to 10 glasses of water daily. Monitor the client's intake and output every hour and administer fluid and electrolyte replacements as ordered. Assess the woman for signs and symptoms of pulmonary edema, such as crackles and wheezing heard on auscultation, dyspnea, decreased oxygen saturation levels, cough, neck vein distention, anxiety, and restlessness. The treatment of acute pulmonary edema is symptomatic and includes the administration of vasodilating agents and of diuretics. The development of acute pulmonary edema in women with hypertension during pregnancy is associated with high levels of IV fluid administration - *Take Note! Preeclampsia increases the risk of placental abruption, preterm birth, intrauterine growth restriction, and fetal distress during childbirth. Always be prepared if you see symptoms of preeclampsia!* - To achieve a safe outcome for the fetus, prepare the woman for possible testing to evaluate fetal status as preeclampsia progresses. Testing may include the nonstress test, serial ultrasounds to track fetal growth, amniocentesis to determine fetal lung maturity, Doppler velocimetry to screen for fetal compromise, and biophysical profile to evaluate ongoing fetal well-being. Other laboratory tests may be performed to monitor the disease process and to determine if it is progressing into HELLP syndrome. These include liver enzymes such as lactic dehydrogenase (LDH), ALT, and AST; chemistry panel, such as creatinine, BUN, uric acid, and glucose; CBC, including platelet count; coagulation studies, such as PT, PTT, fibrinogen, and bleeding time; and a 24-hour urine collection for protein and creatinine clearance. Administer parenteral magnesium sulfate as ordered to prevent seizures. Assess DTRs to evaluate the effectiveness of therapy. Clients with preeclampsia commonly present with hyperreflexia. Severe preeclampsia causes changes in the cortex, which disrupts the equilibrium of impulses between the cerebral cortex and the spinal cord. Brisk reflexes (hyperreflexia) are the result of an irritable cortex and indicate central nervous system (CNS) involvement. Diminished or absent reflexes occur when the client develops magnesium toxicity. Because magnesium is a potent neuromuscular blockade, the afferent and efferent nerve pathways do not relay messages properly and hyporeflexia develops. Common sites used to assess DTRs are biceps reflex, triceps reflex, patellar reflex, Achilles reflex, and plantar reflex. Nursing Procedure 19.1 highlights the steps for assessing the patellar reflex. The National Institute of Neurological Disorders and Stroke, a division of the National Institutes of Health, published a scale in the early 1990s that, although subjective, is used widely today. It grades reflexes from 0 to 4+. Grades 2+ and 3+ are considered normal, whereas grades 0 and 4 may indicate pathology (Table 19.2). Because these are subjective assessments, to improve communication of reflex results, condensed descriptor categories such as absent, average, brisk, or clonus should be used rather than numeric codes. Clonus is the presence of rhythmic involuntary contractions, most often at the foot or ankle. Sustained clonus confirms CNS involvement. Nursing Procedure 19.2 highlights the steps when testing for ankle clonus. With magnesium sulfate administration, the client is at risk for magnesium toxicity. Closely assess the client for signs of toxicity, which include a respiratory rate of less than 12 breaths per minute, absence of DTRs, and a decrease in urinary output (<30 mL/hr). Also monitor serum magnesium levels. Although exact levels may vary among agencies, serum magnesium levels ranging from 4 to 7 mEq/L are considered therapeutic, whereas levels more than 8 mEq/dL are generally considered toxic. As levels increase, the woman is at risk for severe problems: 10 mEq/L: possible loss of DTRs 15 mEq/L: possible respiratory depression 25 mEq/L: possible cardiac arrest - If signs and symptoms of magnesium toxicity develop, expect to administer calcium gluconate as the antidote. Throughout the client's stay, closely monitor her for signs and symptoms of labor. Perform continuous electronic fetal monitoring to assess fetal well-being. Note trends in baseline rate and presence or absence of accelerations or decelerations. Also observe for signs of fetal distress and report them immediately. Administer glucocorticoid treatment as ordered to enhance fetal lung maturity and prepare for labor induction if the mother's condition warrants. - Keep the client and family informed of the woman's condition and educate them about the course of treatment. Provide emotional support for the client and family. Severe preeclampsia is very frightening for the client and her family, and most expectant mothers are very anxious about their own health as well as that of the fetus. To allay anxiety, use light touch to comfort and reassure her that the necessary actions are being taken. Actively listening to her concerns and fears and communicating them to the health care provider are important to keep open the lines of communication. Offering praise for small accomplishments can provide positive reinforcement for effective behaviors.

*Know the causes of a late postpartum hemorrhage.* *TABLE 22.2 FACTORS PLACING A WOMAN AT RISK FOR POSTPARTUM HEMORRHAGE* Clinical Risk Factors & Associated Clinical Conditions [16]

Tone (abnormalities of uterine contractions) 1. Overdistention of uterus: *Polyhydramnios*, Multifetal gestation, Macrosomia 2. Uterine muscle exhaustion: *Rapid labor*, Prolonged labor, Oxytocin use 3. Uterine infection: Maternal fever, Prolonged rupture of membranes Tissue (retained in uterus) 4. Products of conception: Incomplete placenta at birth 5. Retained blood clots: Atonic uterus Trauma (of the genital tract) 6. Lacerations anywhere: Precipitate birth or operative birth 7. Laceration extensions: Malposition of fetus, Previous uterine surgery 8. Uterine inversion: Forceful pulling when placenta isn't separated yet; traction on the cord when uterus isn't contracted Thrombin (coagulation abnormalities) Preexisting conditions: 9. Hereditary inheritance 10. Hemophilia 11. von Willebrand disease 12. History of previous PPH 13. Acquired in pregnancy 14. Idiopathic thrombocytopenia purpura 15. Bruising, elevated BP 16. DIC

dysfunctional uterine bleeding tx idk if this will be on the final or not

Treatment of AUB depends on the cause of the bleeding, the age of the client, and whether or not she desires future fertility. When known, the underlying cause of the disorder is treated. Otherwise, the goal of treatment is to normalize the bleeding, correct the anemia, prevent or diagnose early cancer, and restore quality of life. Treatment options for AUB include combined OCs, progestogens, NSAIDs, tranexamic acid (antifibrinolytic), GnRH analogs, Danazol, and LNG-IUS. Management of AUB might include medical care with pharmacotherapy or insertion of a hormone-secreting intrauterine system. OCs are used for cycle regulation as well as for contraception. They help prevent the risks associated with prolonged, unopposed estrogen stimulation of the endometrium. NSAIDs and progestin therapy (progesterone-releasing IUS [Mirena] or Depo-Provera) significantly decrease menstrual blood loss - If the client does not respond to medical therapy, surgical intervention might include dilation and curettage (D&C), endometrial ablation, uterine artery embolization, or hysterectomy. Surgery should be considered in women for whom medical treatment has failed, cannot be tolerated, or is contraindicated. Endometrial ablation is an alternative to hysterectomy, but both would be for the woman no longer desiring fertility as both procedures can cause infertility. Techniques used for ablation include laser, electrosurgery excision procedure, freezing, heated fluid infusion, or thermal balloon ablation. Most women will have reduced menstrual flow following endometrial ablation, and up to half will stop having periods. Younger women are less likely than older women to respond to endometrial ablation. Recent scientific evidence supports that up to one quarter of clients treated with endometrial ablation require repeat ablation or subsequent hysterectomy to stop AUB. Hysterectomy should be considered a last resort for AUB

*Know the signs and symptoms of hypoglycemia in a newborn* *S/S of hypoglycemia in a newborn*

Tremors jitters irritability cyanosis apnea tachypnea grunting poor tone lethargy seizures high-pitched or weak cry hypothermia poor feeding BS<40

*Understand the intensity of contractions and how to measure them.* Toco: external monitor IUPC Palpate contractions (touch her abdomen; end of nose is mild, *chin its moderate*, forehead is firm)

True Labor- Contraction timing: Regular, becoming closer together, usually 4-6 min apart, lasting 30-60 sec Contraction strength: Become stronger with time, vaginal pressure is usually felt Contraction discomfort: Starts in the back and radiates around toward the front of the abdomen Any change in activity: Contractions continue no matter what positional change is made Stay or go?: Stay home until contractions are 5 min apart, last 45-60 sec, and are strong enough so that a conversation during one is not possible—then go to the hospital or birthing center.

Which assessment would indicate that a woman is in true labor? a. Membranes are ruptured and fluid is clear. b. Presenting part is engaged and not floating. c. Cervix is 4 cm dilated, 90% effaced. d. Contractions last 30 seconds, every 5 to 10 minutes.

c. Cervix is 4 cm dilated, 90% effaced. True labor is characterized by contractions occurring at regular intervals that increase in frequency, duration, and intensity. These contractions bring about progressive cervical dilation and effacement. Thus, a cervix dilated to 4 cm and 90% effaced indicates true labor. Rupture of membranes may occur before the onset of labor, at the onset of labor, or at any time during labor and thus is not indicative of true labor. Engagement occurs when the presenting part reaches 0 station; it typically occurs 2 weeks before term in primigravidas and several weeks before the onset of labor or at the beginning of labor for multiparas. Contractions of true labor typically last 30 to 60 seconds and occur approximately every 4 to 6 minutes.

*Uterine Contractions*

Uterine contractions are involuntary and therefore cannot be controlled by the woman experiencing them, regardless of whether they are spontaneous or induced. Uterine contractions are rhythmic and intermittent, with a period of relaxation between contractions. This pause allows the woman and the uterine muscles to rest. In addition, this pause restores blood flow to the uterus and placenta, which is temporarily reduced during each uterine contraction. Uterine contractions are responsible for thinning and dilating the cervix, then thrusting the presenting part toward the lower uterine segment. The cervical canal reduces in length from 2 cm to a paper-thin entity and is described in terms of percentages from 0% to 100%. In primigravidas, effacement typically starts before the onset of labor and usually begins before dilation; in multiparas, however, neither effacement nor dilation may start until labor ensues (Fig. 13.12). On clinical examination the following may be assessed: Cervical canal 2 cm in length would be described as 0% effaced. Cervical canal 1 cm in length would be described as 50% effaced. Cervical canal 0 cm in length would be described as 100% effaced.

*VAGINAL BIRTH AFTER CESAREAN (VBAC)*

VBAC describes a woman who gives birth vaginally after having at least one previous cesarean birth. Despite evidence that some women who have had a cesarean birth are candidates for vaginal birth, most women who have had a cesarean birth once undergo another for subsequent pregnancies. A multidisciplinary guideline group representing family medicine, epidemiology, obstetrics, and midwifery developed recommendations based on high-quality systematic evidence-based, peer-reviewed research, that individual assessment of risks and benefits be discussed with the pregnant woman with a history of one or more prior cesarean births who are deciding between a planned VBAC or a repeat cesarean birth. A planned VBAC is an appropriate option for most women with a history of prior cesarean birth. Contraindications to VBAC include a prior classic uterine incision, prior transfundal uterine surgery (myomectomy), uterine scar other than low-transverse cesarean scar, contracted pelvis, and inadequate staff or facility if an emergency cesarean birth in the event of uterine rupture is required . Most women go through a trial of labor to see how they progress, but this must be performed in an environment capable of handling the emergency of uterine rupture. The use of cervical ripening agents increases the risk of uterine rupture and thus is contraindicated in VBAC clients. The woman considering induction of labor after a previous cesarean birth needs to be informed of the risks versus benefits with an induction than with spontaneous labor

*laboratory and dgx testing for preeclampsia*

Various laboratory tests may be performed to evaluate the woman's status. Typically these include a CBC, serum electrolytes, BUN, creatinine, and hepatic enzyme levels. Urine specimens are checked for protein; if levels are 1 to 2+ or greater, a 24-hour urine collection is completed.

*BOX 24.3 WITHDRAWAL ACRONYM* *Assess the newborn for signs of neonatal abstinence syndrome using the acronym WITHDRAWAL to focus the assessment:*

W = Wakefulness: sleep duration < 3 hrs after feeding I = Irritability T = Temperature variation, tachycardia, tremors H = Hyperactivity, high-pitched persistent cry, hyperreflexia, hypertonus D = Diarrhea, diaphoresis, disorganized suck R = Respiratory distress, rub marks, rhinorrhea A = Apneic attacks, autonomic dysfunction W = Weight loss or failure to gain weight A = Alkalosis (respiratory) L = Lacrimation

*confidentiality* [documentation on study guide]

With the establishment of the *Health Insurance Portability and Accountability Act (HIPAA) of 1996*, the confidentiality of health care information is now mandated by law. - The primary intent of the law is to maintain health insurance coverage for workers and their families when they change or lose jobs - Another aspect of the law requires the Department of Health and Human Services to establish national standards for EMRs - With the increased use of EMRs and electronic billing came the increased possibility that personal health information might be inappropriately distributed. Client confidentiality and privacy must be maintained in the same manner as it is with paper documentation. Nurses can ensure that privacy is maintained when using computerized documentation and EMRs by doing the following: Always maintain the security of personal log-in information; nurses should never share it with other health care providers or other persons. Always log off when leaving the computer. Do not leave client information visible on a monitor screen when the computer/monitor is unattended. Use safeguards, such as encryption, when using alternative means of communication such as email. Exceptions to confidentiality exist. For example, suspicion of physical or sexual abuse and injuries caused by a weapon or criminal act must be reported to the proper authorities. Abuse cases are reported to the appropriate welfare authorities, whereas criminal acts are reported to the police. In addition, if the minor is a threat to self, then information may need to be disclosed to protect the child. The health care provider must also follow public health laws that require reporting certain infectious diseases to the local health department (e.g., tuberculosis, hepatitis, HIV, and other sexually transmitted infections). Finally, there is a duty to warn third parties when a specific threat is made to an identifiable person. Health care providers must strike a balance between confidentiality and required disclosure. Even if disclosure is required it is recommended that the health care provider attempt to gain consent for the disclosure and when possible inform the minor of the limits to confidentiality and consent prior to the initiation of care

*VAGINAL BIRTH AFTER CESAREAN mgmt*

Women are the primary decision makers about the choice of birth method, but they need education about VBAC during their prenatal course. Management is similar for any women experiencing labor, but certain areas require special focus: 1. Consent: Fully informed consent is essential for the woman who wants to have a trial of labor after cesarean birth. The client must be advised about the risks as well as the benefits. She must understand the ramifications of uterine rupture, even though the risk is small. 2. Documentation: Record keeping is an important component of safe client care. If and when an emergency occurs, it is imperative to take care of the client, but also to keep track of the plan of care, interventions and their timing, and the client's response. Events and activities can be written right on the fetal monitoring tracing to correlate with the change in fetal status. 3. Surveillance: A distressed fetal monitor tracing in a woman undergoing a trial of labor after a cesarean birth should alert the nurse to the possibility of uterine rupture. Terminal bradycardia must be considered an emergency situation, and the nurse should prepare the team for an emergency delivery. 4. Readiness for emergency: According to ACOG (2010) criteria for a safe trial of labor for a woman who has had a previous cesarean birth, the physician or nurse practitioner, anesthesia provider, and operating room team must be immediately available. Anything less would place the women and fetus at risk. Women and their health care providers are advised to consider VBAC in the context of potential risk, available resources, and the health care system. The ACOG (2010) guidelines state that VBACs are safe and appropriate for most women, but emphasize the need for thorough counseling, shared decision making, and client autonomy. Nurses must act as advocates, giving input on the appropriate selection of women who wish to undergo VBAC. Nurses also need to become experts at reading fetal monitoring tracings to identify fetal distress and set in motion an emergency birth. Including all of these nursing strategies will make VBAC safer for all.

When going through the transition phase of labor, women often feel out of control. What do women in the transition phase of labor need the most? a. positive reinforcement b. their significant other beside them c. intense nursing care d. just to be left alone

a. *positive reinforcement*

Which of the following biophysical profile findings indicate poor oxygenation to the fetus? a. Two pockets of amniotic fluid b. Well-flexed arms and legs c. Nonreactive fetal heart rate d. Fetal breathing movements noted

c. Nonreactive fetal heart rate The correct response is C. A nonreactive fetal heart rate is one of the biophysical profile findings that indicate poor oxygenation to the fetus.

*Prolonged decelerations*

are abrupt FHR declines of at least 15 bpm that last longer than 2 minutes, but less than 10 minutes The rate usually drops to less than 90 bpm. Many factors are associated with this pattern, including prolonged cord compression, abruptio placenta, cord prolapse, supine maternal position, vaginal examination, fetal blood sampling, maternal seizures, regional anesthesia, or uterine rupture Prolonged decelerations can be remedied by identifying the underlying cause and correcting it.

*Differentiate the causes and nursing interventions for decelerations:* [VEAL CHOP, know what to do, know what to do for decelerations] o Early o Variable o Late o Prolonged *Early decelerations*

are visually apparent, usually symmetrical, and characterized by a gradual decrease in the FHR in which the nadir (lowest point) occurs at the peak of the contraction. They rarely decrease more than 30 to 40 bpm below the baseline. Typically, the onset, nadir, and recovery of the deceleration occur at the same time as the onset, peak, and recovery of the contraction. They are most often seen during the active stage of any normal labor, during pushing, crowning, or vacuum extraction. They are thought to be a result of fetal head compression that results in a reflex vagal response with a resultant slowing of the FHR during uterine contractions. Early decelerations are not indicative of fetal distress and do not require intervention. Responding to Fetal Heart Rate Distress During Labor: *During possible fetal distress that involves lack of variability, late decelerations, and fetal tachycardia, simply changing the woman's position is inadequate. The nurse should notify the health care provider immediately regarding the situation.*

*Late decelerations*

are visually apparent, usually symmetrical, transitory decreases in FHR that occur after the peak of the contraction. The FHR does not return to baseline levels until well after the contraction has ended. Delayed timing of the deceleration occurs, with the nadir of the uterine contraction. Late decelerations are associated with uteroplacental insufficiency, which occurs when blood flow within the intervillous space is decreased to the extent that fetal hypoxia or myocardial depression exists. Conditions that may decrease uteroplacental perfusion with resultant decelerations include maternal hypotension, gestational hypertension, placental aging secondary to diabetes and postmaturity, hyperstimulation via oxytocin infusion, maternal smoking, anemia, and cardiac disease. They imply some degree of fetal hypoxia. Recurrent or intermittent late decelerations are always category II (indeterminate) or category III (abnormal) regardless of depth of deceleration. Acute episodes with moderate variability are more likely to be correctable, whereas chronic episodes with loss of variability are less likely to be correctable. Box 14.1 highlights interventions for category III decelerations.

*Discuss a laboring patient in the transition phase of labor.* The shortest but most intense phase of labor is the: a. Latent phase b. Active phase c. Transition phase d. Placental expulsion phase

c. *Transition phase* The transition phase of the 1st stage of labor occurs when the *contractions are 1-2 minutes* apart and the final dilation is taking place. The transition phase is the most difficult and, fortunately, the shortest phase for the woman, lasting approximately 1 hr in the first birth and perhaps 15-30 minutes in successive births. Many women are not able to cope well with the intensity of this short period, become restless, and request pain medications. During the transition phase, *the cervix dilates from 8 to 10 cm.*

Several hours later, the client complains of nausea and turns to her partner and angrily tells him to not touch her and to go away. a. What assessment needs to be done to determine what is happening? b. What explanation can you offer Carrie's partner regarding her change in behavior?

i. The nurse should perform a vaginal examination to validate that Carrie is in the transition phase (8 to 10 cm dilated). i. Explain to her partner that she is in the transition phase of the first stage of labor and that her behavior is typical, since she is having hard contractions frequently. Reassure him not to take Carrie's comments personally, but to stay and be supportive to her.

*gonorrhea mgmt*

can be cured with the right treatment - CDC recommends dual therapy (using 2 drugs as the treatment). is recommended to prevent drug resistance and is also effective against chlamydia - Tx of choice for uncomplicated gonococcal infections: *azithromycin 1 g PO in a single dose & ceftriaxone 250 mg IM in a single dose* - *Azithromycin PO or doxycycline* should accompany all gonococcal treatment regimens if chlamydial infection is NOT ruled out - Pregnant women with gonorrhea should NOT be treated with quinolones or tetracyclines. - Pregnant women with a positive test for gonorrhea should be treated with the same recommended dual therapy of *ceftriaxone with either azithromycin or amoxicillin* - To prevent gonococcal ophthalmia neonatorum, a prophylactic agent should be instilled into the eyes of all newborns; this procedure is required by law in most states. - Erythromycin or tetracycline ophthalmic ointment in a single application is recommended. - With use of recommended treatment, follow-up testing to document eradication of gonorrhea is no longer recommended. - Instead, rescreening in 2-3 months to identify reinfection is suggested

*Know and be able to apply the clinical manifestations and nursing management of complications in pregnancy associated with the placenta.* *gestational trophoblastic disease (GTD)*

comprises a spectrum of neoplastic disorders that originate in the placenta! very extreme - There is abnormal hyperproliferation of trophoblastic cells that normally would develop into the placenta during pregnancy! - encompass *hydatidiform mole (complete and partial)*, invasive mole, gestational choriocarcinoma, placental-site trophoblastic tumor, and epithelioid trophoblastic tumor - Gestational tissue is present, but the pregnancy is not viable! - The incidence is hard to determine due to uncommon diagnosis and inaccuracy of documentation of pregnancy loss, but it is thought to occur in about *1 in 1,000 pregnancies in the US*; in Asian countries, the rate is 1 of every 120 - the 2 most common types of GTD are hydatidiform mole (partial or complete) and choriocarcinoma -Most women with a classic complete mole present with vaginal bleeding, anemia, excessively enlarged uterus, *preeclampsia*, and hyperemesis.

*Understand the causes and be able to apply nursing interventions related to labor dystocias.* Dystocia

defined as abnormal or difficult labor, can be influenced by a vast number of maternal and fetal factors. Dystocia is said to exist when the progress of labor deviates from normal; it is characterized by a slow and abnormal progression of labor. It occurs in approximately 8% to 11% of all labors and is the leading indicator for primary cesarean birth in the United States It is of concern because of its fatiguing factor for both mother and fetus and frequently requires medical or surgical interventions, which increases risk.

*Management of placental abruption*

depends on the gestational age, the extent of the hemorrhage, and maternal-fetal oxygenation perfusion/reserve status. Treatment is based on the circumstances. Typically once the diagnosis is established, the focus is on maintaining the cardiovascular status of the mother and developing a plan to deliver the fetus quickly. A cesarean birth may take place quickly if the fetus is still alive with only a partial abruption. A vaginal birth may take place if there is fetal demise secondary to a complete abruption. Treatment of abruptio placentae is designed to assess, control, and restore the amount of blood lost; provide a positive outcome for both mother and newborn; and prevent coagulation disorders, such as DIC. Emergency measures include starting two large-bore IV lines with normal saline or lactated Ringer solution to combat hypovolemia, obtaining blood specimens for evaluating hemodynamic status values and for typing and cross-matching, and frequently monitoring fetal and maternal well-being. After the severity of abruption is determined and appropriate blood and fluid replacement is given, cesarean birth is done immediately if fetal distress is evident. If the fetus is not in distress close monitoring continues, with delivery planned at the earliest signs of fetal distress. Because of the possibility of fetal blood loss through the placenta, a neonatal intensive care team should be available during the birth process to assess and treat the newborn immediately for shock, blood loss, and hypoxia. If the woman develops DIC, treatment focuses on determining the underlying cause of DIC and correcting it. Replacement therapy of the coagulation factors is achieved by transfusion of fresh-frozen plasma along with cryoprecipitate to maintain the circulating volume and provide oxygen to the cells of the body. Anticoagulant therapy (low-molecular-weight heparin), packed red cells, platelet concentrates, antithrombin concentrates, and nonclotting protein-containing volume expanders, such as plasma protein fraction or albumin, are also used to combat this serious condition. The use of blood products must be dictated by the clinical picture and not simply to normalize laboratory test results. Prompt identification and early intervention are essential for a woman with acute DIC associated with abruptio placentae to treat DIC and possibly save her life.

*tertiary prevention* *treat*

designed to reduce or limit the progression of a permanent, irreversible disease or disability • purpose is to restore individuals to their maximum potential. • takes place only if the condition results in a permanent disability. measures are supportive and restorative. • For example, tertiary prevention efforts would focus on *minimizing and managing the effects of a chronic illness such as cerebrovascular disease or the chronic effects of sexually transmitted infections (e.g., herpes, HIV, and untreated syphilis).* • Another example would involve working with *women who have suffered long-term consequences of violence.* • Client education is the cornerstone of all disease management programs. • The focus of the nurse would be to maximize the woman's strengths through education, to help her recover from the trauma and loss, and to build support systems.

Early identification of and prompt interventions for dystocia are essential to minimize risk to the woman and fetus. *factors associated with an increased risk for dystocia include:*

epidural analgesia excessive analgesia multiple pregnancy hydramnios maternal exhaustion ineffective maternal pushing technique occiput posterior position longer first stage of labor nulliparity short maternal stature (less than 5 ft tall) fetal birth weight (more than 8.8 lb) shoulder dystocia abnormal fetal presentation or position (breech) fetal anomalies (hydrocephalus) maternal age older than 34 years high caffeine intake overweight gestational age more than 41 weeks chorioamnionitis ineffective uterine contractions high fetal station at complete cervical dilation

Cold Stress...

excessive heat loss that requires a newborn to use compensatory mechanisms (such as non-shivering thermogenesis & tachypnea) to maintain core body temperature • As temp decreases, newborn becomes *less active, lethargic, hypotonic, & weaker*

*PLATYPELLOID (FLAT) PELVIS*

is the least common type of pelvic structure among men and women, with an approximate incidence of 3% • the pelvic cavity is shallow but widens at the pelvic outlet, making it difficult for the fetus to descend through the mid-pelvis • Labor prognosis is poor with arrest at the inlet occurring frequently • It is not favorable for a vaginal birth unless the fetal head can pass through the inlet. • Women with this type of pelvis usually require cesarean birth

*nursing assessments of postpartum blues* Many postpartum women (approximately 80%) experience the "blues"

experiences rapid cycling mood symptoms during the 1st PP week typically • exhibits mild depressive symptoms of anxiety, irritability, mood swings, tearfulness, increased sensitivity, despondency, feelings of being overwhelmed, difficulty thinking clearly, and fatigue • Emotional lability is the MOST prominent symptom • "blues" typically peak on postpartum days 4 and 5 and usually resolve by postpartum day 10 • Biologic, psychological, and social factors have been hypothesized as relevant to blues causation, but no studies have validated this. • although the woman's symptoms may be distressing, they do NOT reflect psychopathology and usually do not affect the mother's ability to function and care for her infant • usually *self-limiting* and require no formal treatment other than reassurance and validation of the woman's experience, as well as assistance in caring for herself and the newborn • follow-up of women with postpartum blues is important because up to 20% go on to develop postpartum depression

*Rh incompatibility*

exposure of Rh-negative mother to Rh-positive fetal blood; sensitization; antibody production; risk increases with each subsequent pregnancy and fetus with Rh-positive blood Nursing Assessment: maternal blood type and Rh status Nursing Management: RhoGAM at 28 weeks Only will give moms who are Rh- have the potential of getting RhoGam even if she has a miscarriage she will still receive RhoGam Rh+ will NEVER get Rhogam

*nursing assessments of postpartum depression (PPD)* Depression is more prevalent in women than in men, which may be related to biologic, hormonal, and psychosocial factors. It affects as many as 20% of all mothers in the United States, and as many as 60% of adolescent mothers.

form of clinical depression that can affect women, and less frequently men, after childbirth • Unlike the blues, women feel worse over time, and changes in mood and behavior do not go away on their own. • may persist for a minimum of six months if untreated • Different from the baby blues, the symptoms of PPD last longer, are more severe, and require treatment. • It is important for the nurse to provide a listening ear for the woman's concerns and assess the need for a referral for symptoms more than normal baby blues and to assess if psychosis is present S/s: Restless Worthless Guilty Hopeless Moody Sad Overwhelmed Loss of enjoyment Low energy level Loss of libido Cry a lot. Exhibit a lack of energy and motivation. Be unable to make decisions or focus. Lose her memory. Experience a lack of pleasure. Have changes in sleep or weight. Show a lack of concern for herself. Withdraw from friends and family. Have pains in her body that do not subside. Feel negatively toward her baby. Have appetite disturbances. Have feelings of isolation from others. Lack interest in her baby. Worry about hurting the baby. Act detached toward others and infant. Have recurrent thoughts of suicide and death

*HELLP Syndrome!!!*

hemolysis elevated liver enzymes low platelet count - It is a variant of the preeclampsia/eclampsia syndrome that occurs in 10% to 20% of clients whose conditions are labeled as severe - increased risk for complications: cerebral hemorrhage, retinal detachment, hematoma/liver rupture, acute renal failure, DIC, placental abruption and maternal death - It is a life-threatening obstetric complication considered by many to be a severe form of preeclampsia involving hemolysis, thrombocytopenia, and liver dysfunction - Both HELLP and preeclampsia occur during the later stages of pregnancy, and sometimes after childbirth - is a clinically progressive condition - Early diagnosis is critical to prevent liver distention, rupture, and hemorrhage and the onset of DIC - If the condition presents prenatally, morbidity and mortality can affect both mother and baby. - occurs in up to *20% of pregnant women diagnosed with severe preeclampsia* - It is unique, as it is a laboratory-value specific diagnosis. - usually have fewer signs of abnormalities consistent with the metabolic syndrome and a lower prevalence of thrombophilia as compared with preeclampsia women without HELLP - has been reported as early as 17 weeks' gestation, most of the time it is diagnosed between 22 and 36 weeks' gestation - can present prior to the presence of an elevated blood pressure - leads to an increased maternal risk for developing liver hematoma or rupture, stroke, cardiac arrest, seizure, pulmonary edema, DIC, subendocardial hemorrhage, adult respiratory distress syndrome, renal damage, sepsis, hypoxic encephalopathy, and maternal or fetal death - recognition of HELLP syndrome and an aggressive multidisciplinary approach and prompt transfer of these women to obstetric centers with expertise in this field are required for the improvement of maternal-fetal prognosis.

Cindy, a 20-year-old primipara, calls the birthing center where you work as a nurse and reports that she thinks she is in labor because she feels labor pains. Her due date is this week. The midwives have been giving her prenatal care throughout this pregnancy. a. What additional information do you need to respond appropriately?

i. Ask about the frequency and duration of her contractions. ii. Ask about how long she has experienced "labor pains." iii. Ask about any other signs she may have experienced such as bloody show, lightening, backache, *ruptured membranes*, and so forth. iv. Ask if walking tends to increase or decrease the intensity of contractions. v. Ask her when she last felt fetal movement. vi. Ask her how far away (distance) she is from the birthing center. vii. Ask her if she has a support person in the home with her.

*Identify the RN's role and legal responsibilities associated with laboring patients, admission process, documentation, and consent forms.* o Delegation, accountability, how to keep everyone safe *L&D Nurse - provides care to patients in labor who have uncomplicated or complicated deliveries*

i. Providing support for the mother during labor ii. Monitoring the fetal heart rate, and mother's vital signs iii. Measuring the strength and timing of contractions iv. Administering medications and performing diagnostic tests v. Consulting with physicians and other members of the care team vi. Assisting with inducing labor vii. Coaching the mother during delivery viii. Identifying and assisting with complications ix. Assisting with various procedures, including Cesarean sections x. Monitoring and performing tests on newborns xi. Providing education and support to mothers and families after delivery xii. The labor and delivery nurse works in a clinic, hospital or birth center setting, assisting the physician or midwife with all aspects of pregnancy and childbirth. Nursing duties during the birth itself include monitoring fetal heartbeat and the length and strength of contractions, as well as administering medication, coaching the woman and assisting with any complications that arise. Following the birth, they perform tests on the newborn and provides guidance to the mother on all aspects of recovery and infant care. The duties of an L&D nurse may include seeing expectant mothers in their prenatal visits, providing family support and teaching childbirth preparation classes.

*Explain Mastitis and what teaching the nursing would include in her plan of care.* An infection usually on 1 side, red, swollen area, may have chills, fever, flu like symptoms Requires an antibiotic Warm compresses Breastfeeding encourage to empty out her breasts even if she doesn't want to She should breastfeed frequently *Mastitis*

inflammation of the breast causes flu-like symptoms, chills, fever, and malaise. These symptoms may occur before the development of soreness, aching, swelling, and redness in the breast (usually the upper outer quadrant). This condition usually occurs in just 1 breast when a milk duct becomes blocked, causing inflammation, or through a cracked or damaged nipple, allowing bacteria to infect a portion of the breast. Treatment consists of rest, warm compresses, antibiotics, breast support, and continued breast-feeding (the infection will not pass into the breast milk). Explain to the mother that it is important to keep the milk flowing in the infected breast, whether it is through nursing or manual expression or with a breast pump.

*delegation*

involves the transfer of responsibility for the performance of an activity to another individual while retaining accountability for the outcome. i. Used appropriately, delegation can result in safe and effective nursing care. Delegation can free the registered nurse (RN) to focus on assessment and development or revision of the nursing care plan. Delegation allows the RN to attend to more complex patient care needs, develops the skills of nursing assistive personnel, and promotes cost containment for the health care organization. The RN determines appropriate nursing practice by using nursing knowledge, professional judgment, and the legal authority to practice nursing ii. In delegating, the RN must ensure appropriate assessment, planning, implementation, and evaluation. Decision making about delegation is a continuous process. These guidelines provide a quick reference for the delegation decision-making information found in each skill. iii. Look at delegation handout on moodle

*gestational HTN assessment findings*

is characterized by hypertension (>140/90) without proteinuria after 20 weeks' gestation resolving by 12 weeks' postpartum - is defined as systolic blood pressure >140 mm Hg and/or diastolic >90 mm Hg on at least two occasions at least 4 to 6 hours apart after the 20th week of gestation, in women known to be normotensive prior to this time and prior to pregnancy

*ANDROID PELVIS*

is considered the male-shaped pelvis and is characterized by a funnel shape. It occurs in approximately 20% of women • The pelvic inlet is heart-shaped and the posterior segments are reduced in all pelvic planes. • Descent of the fetal head into the pelvis is slow, and failure of the fetus to rotate is common. • The prognosis for labor is poor, subsequently leading to cesarean birth.

*GYNECOID PELVIS*

is considered the true female pelvis, occurring in about 40% of all women; it is less common in men • vaginal birth is most favorable with this type of pelvis because the inlet is round and the outlet is roomy • This shape offers the optimal diameters in all three planes of the pelvis • this type of pelvis allows early and complete fetal internal rotation during labor. • An important principle is that most pelvises are not purely defined but occur in nature as mixed types. Many women have a combination of these four basic pelvis types, with no two pelves being exactly the same. Regardless of the shape, the newborn can be born vaginally if size and positioning remain compatible. The narrowest part of the fetus attempts to align itself with the narrowest pelvic dimension (e.g., biparietal to interspinous diameters, which means the fetus generally tends to rotate to the most ample portion of the pelvis).

*Know and be able to apply the clinical manifestations and management of Preterm Labor.* Preterm labor

is defined as the occurrence of regular uterine contractions accompanied by cervical effacement and dilation before the end of the 37th week of gestation. If not halted, it leads to preterm birth. Preterm births remain one of the biggest contributors to perinatal morbidity and mortality in the world. about 12% of births (one in eight infants) in the United States are premature Preterm birth is one of the most common obstetric complications, and its sequelae have a profound effect on the survival and health of the newborn. The rate of preterm births in the United States has increased 35% in the past 20 years. Preterm births account for 75% of neurodevelopmental disorders and other serious morbidities, as well as behavioral and social problems. They account for 85% of all perinatal morbidity and mortality. In addition, up to $30 billion is spent on maternal and infant care related to prematurity annually Infants born prematurely also are at risk for serious sequelae such as respiratory distress syndrome, infections, congenital heart defects, thermoregulation problems that can lead to acidosis and weight loss, intraventricular hemorrhage, jaundice, hypoglycemia, feeding difficulties resulting from diminished stomach capacity and an underdeveloped suck reflex, and neurologic disorders related to hypoxia and trauma at birth. Many will face the prospect of numerous lifelong disabilities, such as cerebral palsy, intellectual impairment, vision defects, and hearing loss. A recent study's findings indicated that a single course of corticosteroids prenatally improved most neonate's neurodevelopmental outcomes if given before 34 weeks' gestation Although great strides have been made in neonatal intensive care, prematurity remains the leading cause of death within the first month of life and is the second leading cause of all infant deaths The exact cause of preterm labor is not known. Currently, prevention is the goal.

A sinusoidal pattern [not on study guide]

is described as having a visually apparent smooth, sinewave-like undulating pattern in the FHR baseline with a cycle frequency of 3 to 5 bpm that persists for >20 minutes. It is attributed to a derangement of CNS control of FHR and occurs when a severe degree of hypoxia secondary to fetal anemia and hypovolemia is present. It is always considered a category III pattern, and to correct it a fetal intrauterine transfusion would be needed

*Describe pre-procedure education for a hysterosalpingogram.*

is the gold standard in assessing patency (being open and unobstructed) of the fallopian tubes • Fallopian tube obstruction is among the most common causes of female factor infertility • US & MRI are used in this assessment. • 3 to 10 mL of an opaque contrast medium is slowly injected through a catheter into the endocervical canal so that the uterus and tubes can be visualized during fluoroscopy and radiography. • If the fallopian tubes are patent, the dye will ascend upward to distend the uterus and the tubes and will spill out into the peritoneal cavity. FIGURE 4.4. Insertion of a dye for a *hysterosalpingogram* • *The contrast dye outlines the uterus and fallopian tubes on an x-ray to demonstrate patency.* Fig. 7-2. Hysterosalpingography. *Note that the contrast medium flows through the intrauterine cannula and out through the uterine tubes.* Diagnostic tests to determine female infertility may include: *Hysterosalpingography to visualize structural defects* - it's is a procedure where x rays are taken of a woman's reproductive tract after a dye is injected.

A cesarean birth

is the surgical birth of the fetus through an incision in the abdomen and uterine wall and is the most commonly performed surgery in the US. A classic (vertical) or low transverse (horizontal) incision may be used; however, the low transverse incision is more common today. High cesarean birth rates are an international concern. The cesarean birth rate in the US is on the rise at an alarming rate. Today approximately 33% or one in three births occurs this way. This is the 14th consecutive year the cesarean birth rate has risen, despite a number of medical organizations, including the WHO and ACOG, urging medical care providers to work on lowering the cesarean birth rate. Cesarean births may result from maternal, fetal, or placental factors that interfere with a vaginal birth. Several factors may explain this increased incidence of cesarean deliveries: the use of electronic fetal monitoring, which identifies fetal distress early; the reduced number of forceps-assisted births; older maternal age and reduced parity; increasing maternal obesity, with more nulliparous women having infants; convenience to the client and doctor; and an increase in malpractice suits. The leading indications for cesarean births are *previous cesarean birth, breech presentation, dystocia, and fetal distress*. Once a woman has experienced a primary cesarean birth, she has a *90%* chance of having another one in a subsequent pregnancy. Cesarean birth is a major surgical procedure with increased risks compared with a vaginal birth. *The client is at risk for complications such as infection, hemorrhage, aspiration, pulmonary embolism, urinary tract trauma, thrombophlebitis, paralytic ileus, and atelectasis. Fetal injury and transient tachypnea of the newborn also may occur*. Spinal, epidural, or general anesthesia is used for cesarean births. Epidural anesthesia is most commonly used because it is associated with less risk and most women wish to be awake and aware of the birth experience.

*Review the indications for the APGAR scores.*

is used worldwide *to evaluate a newborn's physical condition at 1 minute and 5 minutes after birth* • An additional Apgar assessment is done at 10 minutes if the 5-minute score is less than 7 points. • The HR was found to be the most important diagnostic and prognostic of the 5 signs • It can be used as a *rapid method for assessing the survival of a neonate* • Assessment of the newborn at 1 minute provides data about the *newborn's initial adaptation to extrauterine life* • Assessment at 5 minutes provides a *clearer indication of the newborn's overall central nervous system status* • When the newborn experiences physiologic depression, the Apgar score characteristics disappear in a predictable manner: 1st the pink coloration is lost, next the respiratory effort, and then the tone, followed by reflex irritability and finally HR Take Note! *Although Apgar scoring is done at 1 and 5 minutes, it also can be used as a guide during the immediate newborn period to evaluate the newborn's status for any changes because it focuses on critical parameters that must be assessed throughout the early transition period.*

*Variable decelerations*

present as visually apparent abrupt decreases in FHR below baseline and have an unpredictable shape on the FHR baseline, possibly demonstrating no consistent relationship to uterine contractions. The shape of variable decelerations may be U, V, or W, or they may not resemble other patterns. Variable decelerations usually occur abruptly with quick deceleration. They are the most common deceleration pattern found in the laboring woman and are usually transient and correctable Variable decelerations are associated with cord compression. However, they are classified either as category II or III depending on the accompanying change in baseline variability The pattern of variable deceleration consistently related to the contractions with a slow return to FHR baseline warrants further monitoring and evaluation.

*primary preventions* *promote & prevent*

responsibility of all nurses to incorporate health promotion and disease prevention activities into their professional roles • concept involves preventing the disease or condition before it occurs through health promotion activities, environmental protection, and specific protection against disease or injury. • Its focus is on health promotion to reduce the person's vulnerability to any illness by strengthening the person's capacity to withstand physical, emotional, and environmental stressors. • It encompasses a vast array of areas, including nutrition, good hygiene, sanitation, immunizations, protection from ultraviolet rays, genetic counseling, bicycle helmets, handrails on bathtubs, drug education for school children, adequate shelter, smoking cessation, family planning, and the use of seat belts • approx 3,000 children/yr in US are born with defects of the neural tube - the part of a growing fetus that will become the brain and spinal cord—which can cause severe mental and physical disability or death •*Prevention of neural tube defects (NTDs), such as anencephaly and spina bifida, is an example of primary prevention. The use of folic acid supplementation daily for 3 months before and 3 months after conception reduces the risk of first occurrence of an NTD.* • Women who get enough folic acid have a 50% to 70% reduced risk of having a baby with such a defect. • All women of childbearing age should take 0.4-mg folic acid daily as soon as they plan to become pregnant and should continue taking it throughout the pregnancy to prevent this devastating condition. • Giving anticipatory guidance to parents about poison prevention and safety during play is another example of primary prevention.

*secondary prevention* *screening*

the early detection and treatment of adverse health conditions. • is aimed at halting the disease, thus shortening its duration and severity to get the person back to a normal state of functioning. • *Health screenings* are the mainstay • *Pregnancy testing, blood pressure evaluations, cholesterol monitoring, fecal occult blood testing, breast examinations, mammography screening, hearing and vision examinations, and Pap smears are examples of this level of prevention.* • Such interventions do not prevent the health problem but are intended for early detection and prompt treatment to prevent complications. • If the health condition cannot be cured and further complications and disability moves in, then the tertiary level of prevention is needed.

*Understand the clinical manifestations and management of ABO incompatibility* o Know RhoGAM o Remember mom will be Rh- so baby must be positive *ABO incompatibility!*

type O mothers & fetuses with type A or B blood (less severe than Rh incompatibility) Hemolysis associated with ABO incompatibility is limited to type O mothers with fetuses who have type A or B blood. In mothers with type A and B blood, naturally occurring antibodies are of the IgM class, which do not cross the placenta, whereas in type O mothers, the antibodies are predominantly IgG in nature. Because A and B antigens are widely expressed in a variety of tissues besides red blood cells, only a small portion of the antibodies crossing the placenta is available to bind to fetal red cells. In addition, fetal red cells appear to have less surface expression of A or B antigen, resulting in few reactive sites—hence the low incidence of significant hemolysis in affected neonates. With ABO incompatibility, usually the mother is blood type O, with anti-A and anti-B antibodies in her serum; the infant is blood type A, B, or AB. The incompatibility arises as a result of the interaction of antibodies present in maternal serum and the antigen sites on the fetal red cells.

*biophysical profile (BPP)* nursing implications and pt teaching

uses a real-time US to allow assessment of various parameters of fetal well-being • Fetus responds to central hypoxia by: ↓ movement ↓ muscle tone ↓ breathing • FHR changes: ↓ variability, decelerations • The following criteria must be met to obtain a score of 2, anything < is scored as 0: 1. Fetal breathing movement 2. Gross body movement 3. Fetal tone 4. Nonstress test: FHR variability & reactivity 5. Volume of amniotic fluid (AFI) • Want a score of 8-10 (10 is perfect) • 6 is worrisome • 4 = may try to deliver the baby immediately

*Dysfunctional uterine bleeding* Concern: anemia S/S: no pain, flooding, bleeding between periods, soaking a pad or tampon It's caused by a hormonal imbalance *dysfunctional uterine bleeding s/s* abnormal uterine bleeding (AUB) in the book

vaginal bleeding between periods irregular menstrual cycles (usually <28 days between cycles) infertility mood swings hot flashes vaginal tenderness variable menstrual flow ranging from scanty to profuse obesity acne stress anorexia thyroid disease diabetes complications: *anemia* diagnostics: *CBC*, endocrine studies, ultrasound, endometrial biopsy A thorough history should be taken to differentiate between AUB and other conditions that might cause vaginal bleeding, such as pregnancy and pregnancy-related conditions (abruptio placentae, ectopic pregnancy, abortion, or placenta previa); systemic conditions such as Cushing disease, blood dyscrasias, liver disease, renal disease, or thyroid disease; and genital tract pathology such as infections, tumors, or trauma Signs of polycystic ovary syndrome might be present, because it is associated with unopposed estrogen stimulation, elevated androgen levels, and insulin resistance and is a common cause of anovulation Measure orthostatic BP and orthostatic pulse; a drop in pressure or pulse rate may occur with anemia. The HCP with the nurse assisting, performs a pelvic examination to identify any structural abnormalities.

*Cardiovascular changes during pregnancy* What goes up? HR, Blood volume, CO, Clotting factors BP is NOT normal, that's bad! BV is so increased they get a little diluted In relation to large uterus: suppresses vena cava s/s if suppressed: SOB, dizziness, lips tingling intervention: turn her on her side to reestablish blood flow *cardiovascular adaptations to pregnancy*

• *Slight cardiac hypertrophy R/T increase blood volume and cardiac output* • Apical impulse shifts upward & laterally • *HR increases 10-15 bpm between 14 & 20 weeks gestation* • *Increase in BP is NOT considered normal* • 2nd trimester may have ↓ BP due to vasodilation • Systolic and diastolic murmurs may be heard over pulmonic area • Sinus arrhythmia, premature atrial contractions, and premature ventricular systole (No tx usually required) • *supine hypotension* • 3rd trimester: decrease in blood flow to legs r/t pressure of expanding uterus on veins and arteries (leads to edema and varicosities) • BV Increases 1500 ml or 40%-50% above non-pregnancy levels • BV peaks at 32-34 weeks • physiologic anemia occurs b/c plasma increases faster than number of RBCs (decreased Hgb/Hct) [causes hemodilution] • WBC total ↑ in 2nd trimester • CO increases 30-50% by week 32, in response to increased tissue demand for oxygen • coagulation tendency is greater due to increased in various clotting factors • Fibrinogen increased by as much as 50% • Fibronolytic activity (splitting up or dissolving of a clot) is depressed during pregnancy & PP, Provides protective function to decrease chance of bleeding, but More vulnerable to thrombosis/DVT

*s/s of respiratory distress*

• *The baby's cry reflex stimulates the first big breath at birth!* • RR *>60 or<30!* • Cyanosis • tachypnea • expiratory grunting • *intercostal or subcostal retractions! (sternal retractions)!* • *nasal flaring!* • *see-saw breathing*- chest & abd rise/fall simultaneously; working harder to get air in & out using accessory muscles • Apneic periods lasting more than 15-20 seconds with cyanosis & HR changes require further evaluation *Main 2 signs and symptoms of resp. distress for SATA TEST question?!* 1. *nasal flaring!!* 2. *intercostal or subcostal retractions! (sternal retractions)* Also, 3. Grunting with respirations 4. Rate <30 or >60 breaths/min at rest...warrants further assessment 5. Apneic episodes lasting >20 seconds 6. Tachypnea

APGAR Scoring Each parameter gets a score from 0-2 0= absent or poor response 2= normal response Score should be 8-10 for normal newborn Scores of 4-7 signify moderate distress Scores of 0-3 signify severe distress The higher the score indicates the better condition of the newborn. If 8 points or >, no intervention is needed other than supporting normal respiratory efforts and maintaining thermoregulation. Apgar score is influenced by the presence of infection, newborn maturity, mother's age, congenital anomalies, physiologic immaturity, maternal sedation via medications, labor management, and neuromuscular disorders

• A: appearance (color) cyanotic or pale=0 appropriate body color with blue extremities (acrocyanosis)=1 completely appropriate (pink all over)=2 • P: pulse (HR) absent=0 <100bpm=1 >100 bpm=2 • G: grimace (reflex irritability) no response=0, grimace or frown=1 sneeze/cough/vigorous cry=2 • A: activity (muscle tone) limp or flaccid=0 some flexion/limited resistance to extension=1 tight flexion and good resistance to extension with quick return to flexed position after extension=2 • R: respiratory (respiratory effort) apneic= 0 slow/irregular/shallow=1 regular (30-60 bpm)/strong and good cry=2

*prenatal teaching with a patient with systemic lupus erythematosus (SLE)*

• Autoimmune response in SLE prevent the body from recognizing "self" from "non-self", thus allowing antibodies to be formed that attack the body's own cells and proteins • S/s: swollen joints, extreme fatigue, oral ulcers, skin rashes, and sensitivity to sunlight • Complications: flares of disease activity during pregnancy or in postpartum, preeclampsia, pregnancy loss, miscarriage, stillbirth, fetal growth restriction, and preterm birth • Mgmt: control disease flare-ups, suppress s/s, & prevent organ damage • Tx: NSAIDS, prednisone (Deltasone), and an antimalarial agent, hydroxychloroquine (Plaquenil); Goal is to keep drug therapy to a minimum • Teach: *energy conservation, s/s to report (extreme fatigue, edema, confusion, abdominal pain, wt loss, leg pain, and anorexia), the need for frequent and close monitoring for fetal well-being*

*Breast cancer assessment findings* *Malignant Breast Masses Are Described As*

• Hard to palpation • *Painless!!!* • Irregularly shaped (poorly delineated) • Immobile, fixed to the chest wall • Skin dimpling • Nipple retraction • Unilateral mass • Bloody, serosanguineous, or serous nipple discharge • Spontaneous *nipple discharge (unilateral)* *Most occur in the upper outer quadrant of breast which is important because that's close to the lymph nodes!!!*

*Nägele's rule* to calculate EDD: *determining DOB*

• Identify 1st day of the LMP, count backwards 3 months, then add 7 days Example: • LMP: June 3 • minus three months = March 3 • plus 7 days = March 10 • Or add 7 days to the LMP and count forward 9 mos Example: if the 1st day of the LMP was July 10, 2008, the EDB is April 17, 2009 • Can use an ultrasound but this is the MOST accurate, do this 1st

*Review postpartum hemorrhage nursing interventions.* Tidwell, Postpartum Complications

• Most common cause of PPH is uterine atony • 5 Ts of Postpartum Hemorrhage o *Tone*: uterine atony, distended bladder; boggy o *Tissue*: retained placenta and clots o *Trauma*: vaginal, cervical, or uterine injury o *Thrombin*: coagulopathy (preexisting or acquired) o *Traction*: causing uterine inversion • Mgmt: *immediate fundal massage* *administration of uterotonic medications [Oxytocin (Pitocin), Misoprostol (Cytotec), Methylergonovine maleate (Methergine), Carboprost (Hemabate)]* *maintain primary IV* check v/s *check for bladder distention* prepare the woman for removal of retained placental fragments assess for s/s of shock Take Note! *A soft, boggy uterus that deviates from the midline suggests that a full bladder is interfering with uterine involution. If the uterus is not in correct position (midline), it will not be able to contract to control bleeding.*

*Compare and contrast the various contraceptive methods available and their overall effectiveness - Which method is the most reliable?* Answer on test was *IUD*

• Oral contraception: thickens mucous, preventing penetration of sperm; take at same time every day • Injectable: depo: progesterone, single dose q3months; s/e: decreased bone and mineral density; decreased libido; weight gain • Transdermal: applied weekly for 3 wks, no patch for 4th week; increased risk for VTE and embolism; above 190lb should not use this method • Vaginal ring: good option for women with gastric intolerance or s/e with oral contraceptive • Implantable: delivers synthetic progestin; provides 3yr protection • *IUD: most reliable, highly effective; inhibits sperm and sperm viability* • Emergency: use within 72hrs; should not be used in place of regular BC method; regular BC pills given at a higher dose and more frequently

Cold stress assessment findings [not on study guide]

• Pallor • *mottling* • cool to touch of extremities and abd • Cyanosis • respiratory distress • *Irritability progressing to lethargy (LOW energy level)* • Hypotonia • weak cry/suck • Apnea • bradycardia • *reddish and cold skin* • hypoglycemia • hypoxia • heart arrhythmias Some infants and children may experience chronic hypothermia and show symptoms of: weight loss no weight gains failure to thrive

*Identify measures that would promote coping in a labor patient and a positive labor experience.* Things to help her cope with labor & promote a positive labor experience Reduce her anxiety level Support person with her oClasses or pregnancy exercise programs A 20-year-old primigravida at term, comes to the birthing center in active labor (dilation 5 cm and 80% effaced, -1 station) with ruptured membranes. She states she wants an "all-natural" birth without medication. Her partner is with her and appears anxious but supportive. On the admission assessment, this client's prenatal history is unremarkable; vital signs are within normal limits; FHR via Doppler ranges between 140 and 144 bpm and is regular. a. Based on your assessment data and the woman's request not to have medication, what nonpharmacologic interventions could you offer her? b. What positions might be suggested to facilitate fetal descent?

• Progressive relaxation techniques of locating, then releasing tension from one muscle group at a time until the entire body is relaxed • Visual imagery such as taking a journey in the woman's mind to a relaxing place that is far away from the discomfort of labor • Music to bring about a calming effect as well as a distraction or attention focusing to divert attention away from the laboring process; focusing on sound or rhythm helps release tension and promote relaxation • Massage/acupressure to enhance relaxation, improve circulation, and reduce pain in labor; counterpressure on the lower back to help relieve back pain • Breathing techniques for effective attention-focusing strategies to enhance coping mechanisms during labor • Upright positions such as walking, swaying, slow-dancing with her partner, or leaning over a birthing ball will all enhance comfort and use the force of gravity to facilitate fetal descent. • Kneeling and leaning forward will help relieve back pain. • Pelvic rocking on hands and knees and lunging with one foot elevated on a chair may help with internal fetal rotation and speed a slow labor.


Conjuntos de estudio relacionados

Electromagnetic Spectrum Test Study Guide

View Set

Intro to Clinical: Reactions to Food: Allergies and intolerances - Lifecycle test 3

View Set

Explain the origin of blood cells

View Set

Pregnancy, labor, childbirth, postpartum(uncomplicated) ELSEVIER

View Set

Biology unit five, DNA technology.

View Set